Этого треда уже нет.
Это копия, сохраненная 3 сентября 2016 года.

Скачать тред: только с превью, с превью и прикрепленными файлами.
Второй вариант может долго скачиваться. Файлы будут только в живых или недавно утонувших тредах. Подробнее

Если вам полезен архив М.Двача, пожертвуйте на оплату сервера.
43 Кб, 681x502
18 Кб, 523x199
Тред тупых вопросов #47 #274839 В конец треда | Веб
Тред вопросов о жизни, Вселенной и всем таком.

Спрашиваем то, за что в других местах выдают путёвку в биореактор. Здесь анонимные ученые мирового уровня критически рассмотрят любые гениальные идеи и нарисованные в Paint схемы.

Прошлый тред https://2ch.hk/spc/res/271565.html (М)
#2 #274868
Не знаю зачем, но вкатился.
#3 #274883
Поясните за пикчи.
>>274894>>274904
#4 #274884
Марс внутри теплый?
>>274893
#5 #274893
>>274884
Да, как и любая планета.
#6 #274894
>>274883
В невесомости нельзя ходить, вращая ломом.
>>274896>>274901
#7 #274896
>>274894
Вообще можно. Но достаточных размеров лом не поместиться в консервную банку.
#8 #274901
>>274894
А если кинуть этот лом в туалет МКС, он сломается или МКС сойдет с орбиты?
#9 #274904
>>274883
В оригинале вопрос звучит так:

>Может ли космонавт, быстро вращая тяжелым ломом, изменить ориентацию своего спутника?


Это задача из старого советского учебника то ли по физике, то ли по астрономии.
>>274909
#10 #274909
>>274904
Ну и как? Может?
#11 #274925
>>274909
Если ни за что не держаться, то космонавта самого закрутит в обратную сторону внутри объема спутника. Если сесть в кресло и неподвижно ухватиться, то можно. Только лом придется постоянно крутить, остановишь - и спутник обратно крутанется.
мимо
>>275009
#12 #274940
>>274909
гугли - гиродин
пиндосский вариант - Reaction wheels
может, так при помощи гиростабилизаторов стабилизируют всякие спутники, мкс, даже небо, даже аллаха.
#13 #274955
Зачем космонавту лом в космосе?
#14 #274957
На высоте 400 км температура в районе 500-1000 градусов как они работают в открытом космосе ?
#15 #274961
>>274957
На такой высоте газ такой разреженный, что всем похуй.
>>274972
29 Кб, 402x410
#16 #274962
>>274957

>На высоте 400 км температура в районе 500-1000 градусов

>>274974
#17 #274972
>>274961
Я нихуя не пойму этих изъебок с теплопередачей : типа из-за разреженности атмосферы там нихуя не передается температура или как ? Что тогда ощущать будет рука открытая ? Поясните по нормальному кто-нибудь, хули вы, еба.
>>274974>>274976
#18 #274974
>>274962
А сколько? Все верно жи.

>>274957
>>274972
Ионосфера-с. Бывает и больше. Алсо, кинетическая энергия молекул тоже переходит в тепло при ударе, без малого 8км/сек как-никак, так что нагрев побольше чем тупо теплопередачей. Но на таких высотах по наземным меркам глубокий вакуум, и столкновения с молекулами слишком редки, чтобы заметно нагреть что-либо, собственного излучения вообще почти нихуя нет.
>>274983
#19 #274976
>>274972

>типа из-за разреженности атмосферы там нихуя не передается температура или как ?


Типа того. Имеющийся на такой высоте газ очень горячий, но его так мало и он такой разреженный, что конвекции практически не происходит.

Открытой же руке будет холодно и ОЧЕНЬ больно, потому что с ее поверхности начнет активно испаряться в вакуум влага, а саму руку раздует разницей в давлении.
>>274983
#20 #274983
>>274976
>>274974
Чем тогда отличается температура в 1к градусов цельсия от 20, допустим, на одинаковой высоте ? Она получается, только на процессы в самом газе влияет ?

И еще вопрос: насколько можно приблизиться к Солнцу, не поджарив жопки ?
Спасибо за ответы.
>>274989>>274995
#21 #274989
>>274983
Все зависит от высоты давления, то бишь. На уровне моря 20 градусов будет ощущаться поприятнее, чем 1к, конечно. А на высоте 400 км температура это просто скорость движения молекул, мало влияющая на окружающие твердые предметы.

А поджарить жопку можно и на земной орбите, если не озаботиться отведением тепла. На Луне вон температура поверхности днем может достигать 100 градусов и выше. Жарковато.
242 Кб, 980x522
#22 #274995
>>274983
Температура чего-либо без обмена с горячим/холодным телом для тебя похуй. Так что всё зависит от теплообмена. В космосе тебе нужно знать входящий тепловой поток и сколько тепла ты можешь стравить излучением вовне (поскольку других способов нет, разве только отбрасывать в космос что-нибудь горячее). Если ты их уравняешь - ни холодно, ни жарко тебе не будет. Если одно будет больше другого - будешь нагреваться или охлаждаться. Молекулы окружающего газа не дадут заметного увеличения входящего теплового потока, если конечно ты не на околосветовых скоростях летишь, или не в плотных слоях атмосферы (там выходит глубокий гиперзвук и плазменная феерия)

> насколько можно приблизиться к Солнцу, не поджарив жопки ?


Solar Probe Plus приблизится на 8.5 радиусов Солнца к поверхности. Проектировали вроде что-то, что пролетело бы на трех радиусах, но там уже будет влиять солнечная корона сильно, простым зонтиком не обойдешься.
#23 #275009
>>274925
Почему не хотят на Солнце его ебнуть? Была бы такая-то эпичная вспышка.
>>275010
6 Кб, 279x180
#24 #275010
>>275009
Шта?
#25 #275019
Если бы в космосе был крым он был бы тоже наш, ведь так?
>>275021
#26 #275021
>>275019
Конечно! Где Крым там все наше!
#27 #275036
Наверно было не один раз, но спрошу: чому не отправляют мусор в космос, на Солнце например? Не обычный мусор, который на свалки вывозят - тут понятно, что пиздец дорого наверно, а всякие там радиоактивные отходы етс.
#28 #275044
>>275036
Солнце - самый непрактичный способ утилизации отходов, туда чтобы добраться надо дохуища дельты. Радиоактивные отходы весят пиздец сколько, и могут быть дожжены в будущем. Короче, такого мусора особо и нет, чтобы усираться как в кино.
>>275048
#29 #275048
>>275044
А вот выводить мусор на орбиту, выводить, а потом присобачить какой-нибудь ионный двигатель и пускай пиздорит из солнечной системы пару сотен лет
>>275051
#30 #275051
>>275048
все что на орбите, это уже не мусор, это платина
#31 #275053
Прошу прощения за сверхтупой вопрос, но я не могу понять. Юпитер светится? В смысле излучает в видимом для нас спектре? И еще, на поверхности Плутона темно? Там же почти нет источников света, кроме звезд. Темно как в безлунную ночь в тайге?
>>275082
37 Кб, 1309x836
#32 #275077
Не думаю что в пускотреде это оценят, спрошу/вкину тут.
Идея использовать Эверест как разгонную площадку для РН. Вывод будет осуществляться в трубе под низким давлением. Разгоняющая ступень остаётся на земле, спускается обратно и является многоразовой. Мы преодолеваем самую энергоёмкую часть пути пренебрегая сопротивлением воздуха и весом первой ступени.
#33 #275082
>>275053

> Юпитер светится? В смысле излучает в видимом для нас спектре?


Нет.

> И еще, на поверхности Плутона темно?


http://al-pas.livejournal.com/82083.html

>>275077
Это просто иридий уже, а не платина. Нет, 8км и -30% это очень мало, атмосферные потери даже у РН легкого класса незначительны, тем более по сравнению со строительством ебаного вакуумного поршня длиной 8км на склоне горы проще построить космодром на вершине, блджад
>>275090>>275143
41 Кб, 1024x675
#34 #275090
>>275082
Категория: иридиевые посты спейсача.

>давайте пускать ракету с самолёта


>давайте пускать ракету с горы


>давайте пускать ракету с разгонной петли


>давайте ловить фалькон тросами


>чому фалькон не спускают на парашютах?


>чё там в чёрной дыре?


>космонавты ебутся там на МКС?)0)))0)))0


>а чё буит если в касмас без скафандра выйти?))00)


>а сколька в касмасе градусов?


>а как делают такие фоточки где млечный путь видно кароч?


>м-м-м, терраформация, ахахах, няняня, терраформировать Марс, терраформировать Венеру, терраформировать Энцелад, терраформировать Плутон, терраформировать Землю, терраформировать Юпитер, терраформировать Солнце, терраформировать чёрную дыру (кстати чё там в чёрной дыре?), терраформировать клавиатуру от пекарни, терраформировать мою жопу, терраформировать Аллаха.


Что забыл?
#35 #275093
>>275090
Как срать в невесомости.
Распидорасит ли человека в открытом космосе без скафандра
>>275104>>275130
140 Кб, 504x934
#36 #275104
>>275093

>Как срать в невесомости.


Не снимая свитер.
#37 #275106
>>275090
Лудшебы шапку в тред запилил, наркоман.
>>275108>>275111
11 Кб, 254x254
#38 #275108
>>275106
Кто её читать-то будет блядь, если эти олени и сам вопросотред на нулевой не всегда найти могут.
>>275248
#39 #275111
>>275106

>Лудшебы шапку в тред запилил, наркоман.


Как будто кто-то читает эти сраные шапки.
Оп-хуй
#40 #275114
как то нажравшись беляшей у дяди Ашота, я пришёл домой, и мне стало дико хуёво и я решил вздремнуть. поспал час и проснулся. мне пришла в голову чудная мысль: в мире всё противополжно добро и зло, шаурма и беляшь, свет и тьмя. И тут меня осенило. Есть скорость света, но почему нет скорость тьмы? аноны, ведь возможно такое, что есть скорость тьмы, которая превосходит скорость света?
>>275118>>279547
#41 #275118
>>275114
Ты совсем конченый. Это даже для ликбеза пиздец
>>275121>>279547
#42 #275121
>>275118
а чё такого то
>>276105
#43 #275125
>>274909
Может если космонавт будет не просто вращать ломом, а угрожать своему коллеге и спутнику, в случае если тот не снимет штаны и не даст в пердак.
#44 #275130
>>275093

>Как срать в невесомости.


Так же как и дома, но если поносом, то на реактивной струе межешь улететь вслед за вояджером.

>Распидорасит ли человека в открытом космосе без скафандра


Нет
15 Кб, 1047x738
#45 #275136
Смотрите, если планета будет кубической (предположим что углы под собственным весом не обрушаться), то по ощущениям из-за вектора гравитации казалось бы что куб вогнутый, хотя визуально выглядел бы ровным. А еще бы вся вода в центы сторон стекала.
>>275137
#46 #275137
>>275136
Ты напомнил мне одну "особенность" из KSP: тамошняя взлётно-посадочная полоса идеально ровная, поэтому самолёт, который спавнится в её начале, начинает потихоньку скатываться к центру.
>>275156
#47 #275143
>>275082
Спасибо за ответ.
#48 #275156
>>275137
Это хорошо заметно даже в ИРЛ конструкциях, особенно в длинных зданиях (метров 100 и больше) где отклонение от прямой может измеряться в сантиметрах, и учитывается при конструировании.
>>275157>>276784
#49 #275157
>>275156
о чем ты, все равно фундамент делают, там совсем другие проблемы, а не кривизна земли
#50 #275169
Это правда что только радиация может покинуть чёрную дыру? Если да то почему?
>>275178>>275180
#51 #275178
>>275169
Нет.
>>275179
80 Кб, 619x472
#52 #275179
>>275178
Оправдывайся.
#53 #275180
>>275169
Начнём с того что в дыру ничего не сможет попасть
#54 #275181
>>275179

> по мнению некоторых учёных


> эксперты считают


> британские учёные открыли


> есть мнение


Уебывай
#55 #275185
>>275179
Фотоны блядь не могут, а ебучие альфа-/бета-/гамма-частицы могут? Нахуй и в пизду таких """""""""""""ученых"""""""""""""
#56 #275186
>>275179
Журналисты как всегда перекрутили и обосрались. Radiation вообще-то переводится как "излучение", а не только как "радиация". Черная дыра, в неком смысле, таки излучает, но это скорее тепловое излучение или испарение, а не "радиация."
#57 #275190
>>275179
Либо совсем нихуя, либо ебучие джеты, никакой "радиации". "Радиация" — словечко пятилетних даунов - ебучая желтуха.
347 Кб, 1257x1016
#58 #275196
>>275179
Бля, посоны, это охуенно, это какой-то совершенно новый уровень достоверности пруфа.
Вот вам ещё, оправдывайтесь.
>>275201
#59 #275201
>>275196

> 016М03


Так не вышел ещё, погоди тыщу-другу лет.
>>275234
#60 #275217
>>275216
Зачем космонавту ртуть в космосе? inb4: чтоб топить ломы
#61 #275228
>>275224
Баян про урановые ломы в ртути с башорга - не повод кичиться олдфажностью.
#62 #275234
>>275201
Варпом в прошлое закинуло, мало ли.
#63 #275235
>>275222
спасибо, друг
#64 #275245
>>274839 (OP)
>>275244
Если человек в космосе возьмет что-то тяжелое в руки, гантели или лучше лом и начнет крутиться в невесомости, то какой наивысшей скорости вокруг оси он сможет таким образом достичь? Учитывая что это подготовленный человек с хорошим вестибулярным аппаратом, а не мимокрокодил.
>>275254
#65 #275248
>>275108
Чёт проиграл.

> эти олени и сам вопросотред на нулевой не всегда найти могут.

#66 #275253
>>275090
Почему нельзя быстрее?
Почему с Земли не фоткают флаг на Луне?
Почему планеты по кругу?
Когда варп-драйв?
Когда полетим далеко?
#67 #275254
>>275245
Если лом будет массой с человека, то как будешь крутить, так и будешь вращаться сам (в противоположную сторону). Не учитывая сопротивление воздуха, конечно.
#68 #275255
А если например спутник по орбите земляшки разогнать до почти световой скорости, как он будет наблюдаться с поверхности?
>>275257>>275258
#69 #275257
>>275255
Никак не будет, он улетит в ебеня.
>>275258>>275259
#70 #275258
>>275257
Но перед этим его можно будет наблюдать.

>>275255
Он очебыстро улетит.
>>275260
#71 #275259
>>275257
Это как? Я про не улетать, а чтобы быстро крутился вокруг земляшки под сабатон, не меняя направление от орбиты и не поворачивая, понимаешь?
>>275261
#72 #275260
>>275258

> можно


На самом деле нет, он визуально будет плоским, он не будет ничего отражать, да и время этих "наблюдений"...
#73 #275261
>>275259
Так низзя.
>>275263
125 Кб, 1000x1414
#74 #275263
>>275261
Ну смотри, вот я нарисовал. Большой кружок это земля буква З. Маленький кружок с буквой СП это спутник. Стрелочки это движение вокруг земли. Суть вопроса это будит ли с земли спутник размазываться по небу от быстрой скорости?
>>275265
#75 #275265
>>275263
Он не будет ебашить по кругу, он либо улетит в ебеня, либо его распидорасит в кварк-глюонную плазму
>>275266
#76 #275266
>>275265
Да чего улетит если нос постоянно будет параллельно земли? Ну и разъебать не должно если точно лететь и в землю не врезатся
>>275269>>276787
#77 #275269
>>275266
Ладно, я попытался. Релятивистским сокращением его для тебя сожмёт(визуально(если от него вдруг что-то отразится/излучится)) в линию перпендикулярную движению, НИ ОДИН ЕБУЧИУЙ СЕНСОР не сможет разглядеть настолько тонкий объект с такой скоростью, особенно с релятивистским смещением диапазона. Ты нихуяшеньки не увидишь.
>>275270
#78 #275270
>>275269
То есть он будет совсем нивидимый или как полоска на небе?
>>275271
#79 #275271
>>275270
Совсем-совсем.
>>275272>>275273
#80 #275272
>>275271
Если на один оборот вокруг Земли спутнику потребуется секунда, то его скорость должна быть около 40000 км/с +– . Если он будет за секунду делать оборот вокруг Земли, будет он все таки линией тусклой, м ? Вроде как даже и не близко к световой скорости.
другой долбоеб
>>275274
#81 #275273
>>275271
А если на спутнике будет касмонавт, что он увидит?
>>275274
#82 #275274
>>275272
Че сказать то хотел?
>>275273
Не бывает таких спутников, тебе же пытались объяснить, его центробежкой расхуячит. И космонавта твоего вместе с ним.
>>275276
#83 #275276
>>275274

>Че сказать то хотел?



>Если спутник будет за секунду делать оборот вокруг Земли, будет он все таки линией тусклой, м ?



Конечно же, сопротивлением атмосферы, прочностью спутника и увеличением его орбиты из-за увеличения скорости пренебрегаем.Мы и без тебя знаем, что это невозможно, кеп. Гипотетическая ситуация

Думаю это он имел ввиду, когда спрашивал, как будет выглядеть спутник.
>>275277>>275295
#84 #275277
>>275276
У него не было секунды, было почти световой скорости. В гипотетической ситуации сохранения орбиты спутник распидорасит центробежной силой, тут без вариантов, и без абсолютно_прочного_тела, не та модель чтобы пренебрегать. И релятивистское визуальное изменение длинны как самого об.екта, так и излучаемого/отраженного света тоже нельзя сбрасывать со счетов.
>>275281
#85 #275279
Почему взрывы термоядерных бомб не сдвигают планету с орбиты?
>>275280>>275283
#86 #275280
>>275279
Сдвигают, но очень слабо.
#87 #275281
>>275277

>В гипотетической ситуации сохранения орбиты спутник распидорасит центробежной силой, тут без вариантов, и без абсолютно_прочного_тела, не та модель чтобы пренебрегать.



Что за модель ? Ты зануда или у тебя просто нет воображения ? Хорошо, тогда как изменится длина спутника, у которого скорость 40к км/с, а не 300к ? Не очень-то это близко к световой скорости.
>>275282>>275311
#88 #275282
>>275281
На полтора процента. Примерно. А суть модели в том что масса, разогнанная почти до с и ограниченная орбитой, расползётся по всей орбите и её опять же совсем не будет видно.
#89 #275283
>>275279
Потому что вещество не развивает вторую космическую (атмосфера мешает), и Земля остается замкнутой системой. По той же причине невозможно двигать планету обычными реактивными двигателями, скорость истечения которых меньше второй космической.

А вот гравиманевры аппаратов вокруг планеты сдвигают её с орбиты. И удары метеоритов.

Вообще, если подумать - если крышка во взрыве Pascal-B таки покинула атмосферу на скорости больше второй космической, и не испарилась нахуй, то возможно она немного сместила планету с траектории, действуя как рабочее тело реактивного движителя.
https://en.wikipedia.org/wiki/Operation_Plumbbob#Propulsion_of_steel_plate_cap
Алсо, были же высотные ядерные испытания, в космосе - возможно остатки плазмы, разогнанные до бешеных скоростей, тоже покинули планету. (твердого там ничего не было)
>>275307>>275399
#90 #275295
>>275276

>сопротивлением атмосферы, прочностью спутника и увеличением его орбиты из-за увеличения скорости пренебрегаем


Просто скажи, что ты хочешь увидеть, и спроси, чем еще нужно пренебречь, чтобы было так, как ты хочешь, епта.
#91 #275307
>>275283
66 км, в 4 раза выше третьей космической
>>275308>>275309
#92 #275308
>>275307
Пару секунд и на НОО. Но она в плазму скорее всего обратилась.
>>275345
#93 #275309
>>275307
Если не испарилась нахуй и/или не замедлилась, о том и речь
>>275345
#94 #275311
>>275281

>Ты зануда или у тебя просто нет воображения ?


Пиздец просто! Я понимаю, ты зашел в тред тупых вопросов. Но надо ведь понимать, что если ты получаешь ответ, то дает его не такой тупой человек, как ты. Человек, оперирующий современными представлениями и постулатами официальной науки. Если же тебе нужны товарищи по теории о радужных единорогах, танцующих ламбаду на двух копытах под горизонтом событий, то поищи другой раздел. Какой-нибудь аналог /bb.
>>275312
#95 #275312
>>275311
Но ты действительно, какой то деревянный, есть такое, вполне валидное понятие, как мысленный эксперимент. Это один из инструментов познавательных наук.

>В гипотетической ситуации сохранения орбиты спутник распидорасит центробежной силой


В твоей модели я рекомендую тебе подумать о том, как этот спутник будет сохранять орбиту, и уже после этого переходить к утверждениям распидорасит или нет.
Запомнил? Только в такой последовательности. пися в писю - хорошо, пися в попу - плохо
другой
>>275315
#96 #275315
>>275312

>мысленный эксперимент


Мысленный эксперимент, отвергающий всякие ограничения, предусмотренные современной физикой, не более ценен, чем наркоманский трип молодого объебоса.

>В твоей модели


Я никаких моделей не приводил здесь.
>>275336
#97 #275327
>>275220
Интересно, Юнона не будет это изучать?
#98 #275336
>>275315
те выводы относительно распидорасит, это какраз пример такого трипа?
Я полностью согласен с этим выводом, в существенной части вариантов, это логичный вывод, но в них же и такой волшебной скорости, без поддержки сил зла и некромантов, не будет. Если мы делаем одно допущение, почему и в первую очередь Зачем, мы не делаем другого допущения, которое могло бы придать этой ситуации какой то смысл.

>Мысленный эксперимент, отвергающий всякие ограничения, предусмотренные современной физикой, не более ценен


Товарищ Максвелл с тобой не согласен, товарищ Шредингер с тобой не согласен, товарищ Галилей, я не буду говорить про самого главного товарища прошлого столетия и тд.
В качестве предположения, в котором Я более чем уверен, товарищ Лобачевский с тобой тоже не согласится. не уверен права что там обошлось без, явно молодые пёзды были вовлечены, не знаю как с остальным в то время.

Вопрос изначально тупой, но шик в элегантном ответе, а не в сворачивании тензора по всем индексам.
#99 #275337
>>275220
Там прост местным жителям холодно становится они включают ритэги на полную ну вот и греется я думаю
>>275343
185 Кб, 600x810
#100 #275343
>>275337

>местным жителям

>>275350
#101 #275345
>>275308
>>275309
Один из моментов который и дает повод для размышлений, ядерный взрыв эта конструкция перенесла, и продолжала оставаться все еще даже твердой.

Те это подумать только, она пережила нечто, что ускорило ее до 66кмс за, сколько там простите длилось ускорение ?

Сточило ее потом за 10 сек или нет, это вопрос. Если первые 20км пережила, то могло и не сточить.
Но вообще я за повтор, боеголовки то утилизировали, надо было повторить.
>>275350
#102 #275350
>>275345
Осталось лишь отменить договор о запрещении испытаний ядерного оружия. Хотя можно к лучшей корее обратиться им похуй
>>275343
>>275346
Ну там возможно целая планета плавает так что хз
>>275362
#103 #275362
>>275350

>К настоящему времени Договор подписало 183 государства, в том числе 41 государство из 44, необходимых для вступления договора в силу — не подписали только «молодые» ядерные державы Индия, Пакистан, КНДР.



>Ратифицировали Договор 164 государств, в том числе 36 государств из 44, необходимых для его вступления в силу — помимо не подписавших ядерных держав его не ратифицировали другие ядерные державы: США, КНР, Израиль, а также Иран, подозревающийся в разработке ядерного оружия, и Египет.



>6 декабря 2006 Генеральной ассамблей ООН была принята резолюция, подчёркивающая необходимость скорейшего подписания и ратификации Всеобъемлющего договора о запрете ядерных испытаний. За принятие резолюции проголосовали 172 страны, против — две: КНДР и США.



РФ подписало его в 2000 году, но я так понимаю в силу он так за 20 лет и не вступил.
Те все кто мог, все и могут, и это ради космоса
>>275365
#104 #275365
>>275362
Никто не даст засирать земляшку изотопами даже ради космоса
>>275370
#105 #275370
>>275365
никто и никогда, да да знаем.
#106 #275399
>>275283
То есть запуск ракет сдвигают орбиту?
>>275404>>275414
#107 #275404
>>275399
Нет, они же не передают импульс Земле.
>>275414
#108 #275414
>>275404
для передачи надо пнуть ее чтоли?

>>275399
в меньшей степени, чем то говно которое сыпется на землю из космоса и давит солнце.
#109 #275523
Предположим, есть цилиндрический космический аппарат в вакууме. На одно конце цилиндра — солнечный парус. На другом — мощный лазер, который может разогнать аппарат с парусом. Включаем лазер и направляем на лазер. Аппарат же не начнёт ускоряться? А почему?
#110 #275524
>>275523
Потому что нельзя вытащить себя из болота за волосы
#111 #275525
У Вселенной есть край? Тоесть теоретически существует самая крайняя звезда, по одну сторону от которой нет больше других звезд?
>>275534
#112 #275531
Аноны, сколько может стоить ракета выводящая на НОО 100кг? Есть ли спрос сейчас на такие штуки?
>>275535>>275536
#113 #275533
>>275523
Если убрать парус, засунуть всю систему в ведро с авпаянным верхом, заменить лазер на магнетрон и облучать магнетроном меньшее дно ведра, то тогда можно.
#114 #275534
>>275525

Нет
>>275544
#115 #275535
>>275531
Дополню. Имею желание написать небольшую книгу о человеке который с детства мечтал строить ракеты, и реализовал свою мечту построив коммерчески успешный носитель лёгкого класса. Такой маск по-русски.
Вот что интересует:

> примерная стоимость разработки лёгкой рН


> какая схема пуска лучше: традиционный, воздушный старт?


> какое топливо лучше подходит?


> реально шли частной компании договорится например с правительством конго об аренде земли под космодром?


Ну пока все, потом ещё поспрошаю
#116 #275536
>>275531

> НОО 100кг?


От тещи хочешь избавиться?
Сомневаюсь, что есть такие маленькие ракеты. Скорее просто твою посылку покладут в контейнер большой ракеты вместе с другими грузами/спутниками, которые оптом будут выводиться на орбиту.
>>275537
#117 #275537
>>275536
Мне кажется долго ждать придётся. А вдруг надо какая-то конкретная орбита?
#118 #275538
>>275523
Начнет разгоняться,
несколько похуже, чем если просто выкинуть этот парус и светить лазором.

Более того, если один конец этого корыта более теплый чем другой, тоже начнет разгоняться.
>>275539
#119 #275539
>>275538
С хера ли ему разгоняться? Скажи ещё если в парус на лодке подкть то поплывешь.
>>275552>>275597
#120 #275544
>>275534

Но почему нет? Ведь Вселенная конечная и предположительно плоская, а значит не закольцована, тогда почему нет края?
>>275573
14 Кб, 640x400
#121 #275552
>>275539
както так пик

про подуть парус лодка - посмотри эту молодеж мифбастерскую, у них было, движение зафиксировали
#122 #275553
>>275552
Такой бред даже не гуглится
>>275673
#123 #275554
>>275552

> разрушители


> пруф


Ты к нам из бэ заглянул?
#124 #275573
>>275544

>Вселенная конечная


не в том понимани, которое ты вкладываешь в это. Она имеет определенный объем в какой-то момент времени, но при этом остается бесконечной

>плоская


нет

>не закольцована


нет
>>275576
#125 #275575
>>275552
Тебе не кажется, что парус в этой схеме нихуя не даёт лишний? Если ЛАЗОР ТОЛКАИТ, тогда что мешает напрямую получать от него ускорение без всяких парусов и двойного отражения?
>>275676
sage #126 #275576
>>275573

>плоская


>нет


Вообще-то да.
>>275582
#127 #275582
>>275576
Есть разные мнения на этот счет. Если хочешь считать, что она плоская - ок, считай так. В интернете даже "пруфы" есть. Только ответь, что собой представляет край вселенной и что находится за ним?
>>275589>>275847
#128 #275585
>>275090

>чому фалькон не спускают на парашютах?


Кстати, я давно хотел спросить. Почему?
sage #129 #275589
>>275582

>В интернете даже "пруфы" есть.


В интернете есть научные статьи на эту тему.

>что собой представляет край вселенной


Ты какой-то имбецил, плоскость вселенной - это топологическая характеристика, при чем тут некий "край"?
>>275590>>275613
#130 #275590
>>275589
Правильно! И Земля плоская, ведь это тоже топологическая характеристика! Я давно знал!
19 Кб, 300x100
#131 #275597
>>275539

>Скажи ещё если в парус на лодке подкть то поплывешь


Ээм, а почему бы и нет? Движение будет соответственно микроскопическим, но что тут по твоему противоречит физике? Это же не пикрилейтед все же.
>>277456
#132 #275613
>>275589

>В интернете есть научные статьи на эту тему.


Я о них и говорил - разные статьи по разным теориям.

>Ты какой-то имбецил, плоскость вселенной - это топологическая характеристика, при чем тут некий "край"?


Ты тупой или тупой? Если она не замкнута, а "плоская", то это значит, что она не имеет постоянного объема и что летя в одну сторону ты не вернешься в точку старта с другой стороны. В таком случае плоская вселенная либо имеет край либо тупо бесконечна в прямом смысле этого слова. Но только нам известно, что она расширяется и возникла когда-то в прошлом. Тогда как бесконечнойсть без края могла сразу возникнуть, не расширяясь в конкретных размерах?
>>276107
65 Кб, 1305x648
#133 #275630
Тащи космонавты, я задался вопросом, как будет проходить межзвездный перелет для фантастического судна, способного развивать ускорение в 500 g. С механикой движения тел в невесомости знаком отдаленно, но кое-что знаю. Пик может дать вам представление о моем уровне подготовки. Так, по моим предположениям, корабль будет совершать свое путешествие. Насколько это соответствует действительности? Есть ли материалы, с которыми мне надо ознакомиться, может формулы, таблицы или специальные проги, которые помогут рассчитать скорость и ускорение корабля?
>>275636>>275688
971 Кб, 624x649
#134 #275636
>>275630

>как будет проходить межзвездный перелет для фантастического судна, способного развивать ускорение в 500 g


>500 g


>как будет проходить


Примерно как на пикрилейтед. постоянного ускорения в 1g йобалёту более чем хватит для быстрого набора практически любой скорости.

>формулы, таблицы или специальные проги, которые помогут рассчитать скорость и ускорение корабля?


Путь, скорость и ускорение рассчитываются по учебнику 7 класса. Но если ты до глубоко релятивистских скоростей собрался разгоняться, то всё значительно сложнее, алсо пока будешь лететь, звездяшки на месте стоять тоже не будут.
>>275699
283 Кб, 752x642
#135 #275673
#136 #275676
>>275575

>Тебе не кажется, что парус в этой схеме нихуя не даёт лишний?


мне не просто кажется, я об этом прямо написал, вопрошающему оратору, но нашлось какое то дерево и пришлось рисовать картинку - 15 минут убил.
#137 #275677
Можно ли на какой-либо из современных ракет отправить что-то прямиком на Солнце? Или единственный вариант - это сманяврировать вокруг Юпитера/Сатурна?
>>275680>>275745
#138 #275680
>>275677
НИЧОСИ, оказывается, я платиновый вопрос задал (даже ИТТ что-то похожее есть >>275036)

Тем не менее, можно или нет?
>>275701
55 Кб, 880x501
#139 #275688
>>275630

>уровне подготовки.


пейнт освоен, садись, три

откуда эта синусоида ускорения, почему она синусоида.
>>275699
412 Кб, 2048x2048
#140 #275693
С какого расстояния сделали эту фотографию Млечного пути? Каким аппаратом?
>>275694>>276228
#141 #275694
>>275693
хабл, это знать надо!
>>275778
#142 #275699
>>275636
Я понимаю, что человека размажет от таких перегрузок. Предполагается, что экипаж сможет такое ускорение пережить. С помощью каких-нибудь генераторов гравитации или еще что. Это уже отдельный вопрос. Я просто хочу представить такую ситуацию, когда рентабельны коммерческие межзвездные грузоперевозки.

>>275688
А чем синусоида не нравится? Это схематичный график, показывает, как должны изменяться скорость и ускорение во время полета. Реальный график действительно может не быть таким, я просто общий характер зависимости изобразил.

Я бы хотел получить представление о наиболее приемлемом способе доставлять грузы от звезды к звезде. Что будет лучше - несколько чередующихся разгонов и торможений, быстрый разгон и медленное долгое торможение, медленный разгон и быстрое торможение? Нужны ли мощные тормозные двигатели, или вообще можно круто погасить скорость гравитационными маневрами уже прибыв в планетарную систему? Или можно в межзвездном пространстве развернуть маневровыми двигателями корабль кормой вперед и тормозить с помощью маршевых? Как можно решить задачу максимально дешево и быстро преодолевать межзвездные расстояния?
>>275702>>275740
#143 #275701
>>275680
можно, но долго и мало
#144 #275702
>>275699

>А чем синусоида не нравится?


механизмом стоящим за разгоном.
В первую очередь мне скорость не понравилась, при такой зависимости ускорения, сильно неправильно нарисовано.

с остальным в сайфай, там три треда на эту тему, на первой.
и еще пара тредов было израсходовано до этого, еще могло и не утопнуть.
я к тому что это пиздец обширная тема.
>>275733
#145 #275733
>>275702
добавлю
рекомендую тред Экономика голактической империи. , в обязательном порядке, отличный тред получился
#146 #275740
>>275699

>Предполагается, что экипаж сможет такое ускорение пережить.


Это тебе в /sf/.

> Я просто хочу представить такую ситуацию, когда рентабельны коммерческие межзвездные грузоперевозки.


Всё что дальше гипотетической возможности банально туда долететь - это тоже епархия /sf/. Там всё это есть.

> Что будет лучше - несколько чередующихся разгонов и торможений, быстрый разгон и медленное долгое торможение, медленный разгон и быстрое торможение?


Это настолько дурацкий вопрос даже для этого треда, что оптимальней посоветовать почитать Левантовского, иначе такой пиздец и дальше будет возникать в твоей голове.
#147 #275745
>>275677

>Можно ли на какой-либо из современных ракет отправить что-то прямиком на Солнце?


Вряд ли. Если только совсем мало, то в принципе на малой тяге можно (благо электричества для ЭРДУ будет море), и ждать придется до охуевания. Другой вопрос нахуя это надо, если можно сманеврировать.
#148 #275778
>>275694
Совсем пизданулся?
#149 #275836
>>275090

>Зачем в космос людей посылать?


забыл
#150 #275840
Если массивный космический лифт Земли сломается и упадет, сила взрыва будет большая и на большой площади?
>>275857
60 Кб, 600x450
#151 #275847
>>275582

>Только ответь, что собой представляет край вселенной и что находится за ним?



В незамкнутой вселенной край представляет собой именно КРАЙ, за которым нету нихуя ни звезд ни темной материи ни какой либо иной, даже пространства нету. Но нам "повезло", что сфера нашей Наблюдаемой Вселенной вроде как край не затрагивает(как и красный треугольник на пикче). Так что нам не дано этот край увидеть ни при каких раскладах, равно как и вообще выбраться за пределы Наблюдаемой Вселенной или узнать что или кто там находится.

Но гипотетически то он существует. В научпопулярных фильмах расширяющуюся Вселенную любят изображать как надувающийся воздушный шарик. Соответсвенно край Вселенной - поверхность этого шарика, но мы живём внутри него и до краёв нам не добраться, так как они ускользают от нас быстрее скорости света. Развечто через вормхолы
#152 #275857
>>275840
Во-первых: где он сломается? Во вторых: как далеко простирается лифт от Земли?

1: если сломается у основания, то он просто улетит нахуй в космос из-за инерции. Ну может какая-то докосмическая часть ебанётся на планету и ёбнет как следует.
#153 #275862
Анон, смотри, что написано на вики

> Температура короны — порядка миллионакельвинов. Причем от хромосферы она повышается до двух миллионов на расстоянии порядка 70000 км от видимой поверхности Солнца, а затем начинает убывать, достигая у Земли ста тысяч кельвинов


И даже ссылка на какую-то книгу. Книги у меня нет, но стало интересно, как так? Почему тогда наши спутники не сгорают во всеочищающем огне?
156 Кб, 800x1203
#154 #275863
>>275862
Если завернуть в фольгу, то норм будет. А если поджарить солнечную панель, то можно даже электричество получить!
3448 Кб, 1936x1576
#155 #275865
>>275862
Или вот еще. Станция Скайлэб. В свое время американцы проебались с теплоизоляцией станции. То ли отвалилось у них там что-то, то ли треснуло - лень сейчас искать. В результате внутри стало ПРИПЕКАТЬ. Поэтому астронавты повесили над проебанным местом зонтик из фольги (желтый прямоугольник в ближней части станции). И стало норм.
#156 #275878
>>275862
Потому что корона очень разрежена и на нашем удалении представляет собой солнечный ветер. т.е. редкие ионы, летящие от солнца. Десятки тысяч столкновений с молекулами в секунду (условно говоря) ты даже не заметишь, пусть там хоть миллион градусей будет.

Алсо, на низких орбитах даже от этого мизера защищает магнитосфера.

Опасность для спутников представляет скорее электричество от них - долбящие ионы его потихоньку заряжают, как конденсатор.
#157 #275882
Вкачусь, пожалуй.
В принципе, нам уже известно как эволюционируют звёзды. Соответственно вопрос: возможно ли предсказать, когда та или иная звезда "бахнет" сверхновой, и наблюдались ли звёзды, которые вскоре могут ими стать?
>>275886
#158 #275886
>>275882
не давно только первое наблюдение было перехода из пред- в сверхновую
в новостном треде вроде
но я бы сказал минут за пять можно уже предсказать
>>275938
#159 #275888
Вот ученые говорят, мы построили Хаббл, теперь можем заглянуть в ПРОШЛОЕ, это значит, что Хаббл улавливает свет от далеких звезд, но разве свет многих "умерших" звезд не мог за миллиарды лет пролететь мимо нас? Это значит спутники работают по пронципу что уловим, то и покажем? Еще учОные говорят мы должны построить спутник ЕЩЕ БОЛЬШЕ чтобы заглянуть ЕЩЕ дальше в прошлое, значит ли это, что они смогут увидеть к примеру ту же звезду в более раннем ее состоянии? Исходя из всего этого, получается, что мы видим прошлое звезд и галактик, а это значит, что мы никогда не увидим "настоящего" звезд? Вдруг там рептилоиды прямо сейчас транспортируют звезду в свою галактику?
>>275892>>275901
#160 #275892
>>275888
да все именно так
>>276113
#161 #275899
>>275847
Хуита какая-то. Отличить замкнутое многообразие с краем и без края сможешь? Всегда ли незамкнутые многообразия не имеют края (и почему)? И вообще, чего вы полезли копаться в топологии в треде для даунов?
>>275903>>276116
#162 #275901
>>275888

>значит ли это, что они смогут увидеть к примеру ту же звезду в более раннем ее состоянии?


Нет.

>ЕЩЕ БОЛЬШЕ чтобы заглянуть ЕЩЕ дальше в прошлое, значит


Значит что они смогут заглянуть на большее расстояние, а значит на более раннее время.
>>276113
#163 #275903
>>275899

>чего вы полезли копаться в топологии в треде для даунов?


Это уже традиция.

>возможно ли предсказать, когда та или иная звезда "бахнет" сверхновой


Может сто лет назад бахнула, а может еще 100000 проживет, но масштабах вселенной - это небольшой срок. Либо, если удалось поймать совсем предсмертное состояние, когда уже вот через 5 минут. Или или. Предсказать, что бахнет через 10 лет, например, нет возможности.
>>275938
247 Кб, 1280x1206
#164 #275911
Я хочу знать все о скафандрах. Подробный принцип работы, как его одевают (например я так и не нагуглил как именно надевают EMU) и т.п.
>>275919>>276045
#165 #275915
>>275847
Где-то читал, что доказали, мол не замкнутая. Мол была бы замкнутая - было бы видно по распределению реликтового излучения
>>275916
#166 #275916
>>275915

>Где-то читал, что доказали, мол не замкнутая. Мол была бы замкнутая - было бы видно по распределению реликтового излучения


https://ru.wikipedia.org/wiki/Критическая_плотность_(космология)
#167 #275918
Что представляет собой фундаментальные частицы? Имеют ли они конкретную форму? Есть нормальные модели в трёхмерном пространстве? В чем отличие, например, кварка от лептона, если мы смогли бы их увидеть?
#169 #275938
>>275886
И впрямь, нашёл. Спасибо анон!
>>275903
Ага, понял, спасибо.
24 Кб, 200x338
80 Кб, 433x650
195 Кб, 1080x1252
18 Кб, 250x388
#170 #276045
>>275911

>как его одевают


У него сзади дверь. Стучишься в нее и тебе открывают.
>>276085
#171 #276085
>>276045
Там памперс штоле
>>276089
#172 #276087
Пацаны, не бейте, лучше обоссыте.
Я вот методом гугла пытаюсь почуть учить астродинамику. И я немношк охуеваю от сложности математического аппарата. Давайте нехуёво так проинтегрируем вот эту хуиту и получим другую хуиту, а дельта-вэ получается из диффура с удельной мощностью реактивного двигателя по времени с массой нагрузки блаблабла.
Но всякие симуляторы небесной механики и астродинамики (я про игрушки с реализмом много выше среднего KSP, Orbiter, тыры пыры) ведь явно не решают интегральные и дифференциальные уравнения в реальном времени - если даже матлаб тормозит, когда их решает, то движок игры должен просто супер-тормозить. Как же тогда там всё моделируется? Почему этого хер найдёшь в гугле? Что на эту тему почитать?
>>276094
#173 #276089
>>276085
Да. На случай если вдруг кто-то постучится в твою дверь в космосе. Когда все остальные члены экспедиции внутри.
>>276393
#174 #276094
>>276087
Решают. Тебе нужны Рунге-Кутта (просто) и симплектические интеграторы (сложно). А вообще численные методы - это очень обширная область.

Все дело в требуемой точности и других ограничениях, накладываемых на численное решение, алсо матлабы и питоны это хуитки для прототипирования, а не конечный оптимизированный софт. Орбитер кроме того хуярит в реалтайме мало тел, хоть там учитывается довольно много мелкоэффектов типа неравномерности гравитации и давления света, т.е. там не тупо классический n-body. Вот когда ускорение времени включишь - охуеешь. Там начнёт упрощаться всё. И GMAT какой-нибудь тоже строит быстро траектории малого кол-ва тел.

> И я немношк охуеваю от сложности математического аппарата.


Вся механика такая. Ты охуеешь сколько нужно, чтобы заставить летать простейший квадрокоптер. Да и физика, по большому счету.
>>276097
#175 #276097
>>276094
Спасибо анон. В целом, я подозревал что решения просто аппроксимируются, но думал что есть некоторые упрощённые формулы расчёта этого всего, для не слишком реалистичных (но и не аркадных) игр.
#176 #276105
>>275121
с какой скоростью ты движешься, когда стоишь на месте (в ИСО, т.е твое тулово по отношению к какой-либо точке на земной поверхности, например под твоей левой пяткой)? надеюсь, аналогия понятна?
>>276111
#177 #276107
>>275613

>Но только нам известно, что она расширяется и возникла когда-то в прошлом



>нам известно


Демиург внутреннего круга среди нас!
Возьми на Мизру, позязя. Ток на синий материк.
#178 #276111
>>276105
Нет
>>276114
#179 #276113
>>275901
>>275892
Спасибо.
#180 #276114
>>276111
траллишь чтоль? еще проще: с какой скоростью ты двигаешься в заглушенном автомобиле?
>>276130
#181 #276116
>>275899

>Отличить замкнутое многообразие с краем и без края сможешь? Всегда ли незамкнутые многообразия не имеют края (и почему)? И вообще, чего вы полезли копаться в топологии в треде для даунов?


Несмогу.
Потому-что я даун-гуманитарий, но мне кажется охуенно важным представить как выглядит Вселенная в целом, в рамках общепринятых теорий.

Я непонимаю, как конечное количество звёзд(материи) может быть бесконечным?
В случае с замкнутой вселенной всё понятно, а в случае с плоской?
Возможно ли это изобразить на пикчах или объяснить метафорами? На педивикии в статье про многообразия ни одной пикчи и дохуя непонятных слов и символов, которые ломают мне моск
>>276127
#182 #276127
>>276116
такую бесконечность можно представить, например, так:
на краю Вселенной есть звезда. она не совсем обычная - ее западное полушарие красное, а восточное синего цвета (опустим причины сего). теперь представь, что астрономы из США навели телескоп в самую далекую точку и они увидели звезду синего цвета. одновременно с этим, астрономы из России навели свой телескоп, так же в самую дальнюю точку, но с востока и увидят ту же звезду, но с другой стороны и уже красного цвета. таким образом - все замкнуто, но отсутствует понятие "центр". центр в той точке, где находится наблюдатель
>>276128
#183 #276128
>>276127

Ну это, как я понимаю, описана замкнутая Вселенная. В ней если всё время лететь в одну сторону то когда-нибудь вернешься в исходную точку. Таким образом замкнутую Вселенную можно представить как змею схватившую себя за хвост, и тут вроде бы понятно откуда бесконечность берется....

Но в незамкнутой вселенной ведь невозможно увидеть одну и ту же звезду, емли смотреть в разные стороны?
А по последним данным(пруфы приведены выше), ученые склоняются что Вселенная именно незамкнутая. Но при этом бесконечная. Но с кончным числом материи. Вот это я догнать немогу.
>>276131>>276137
#184 #276130
>>276114
У меня нет автомобиля
>>276132
#185 #276131
>>276128
да, догнать тут будет сложно без обмазкой мат.аппарата. аналогии/метафоры/абстракции трудно привести, т.к это всегда сравнение, а в данном случае, у людей, даже и не с чем сравнивать, все на интуитивном уровне и на кончике пера.

в моем понимании "замкнутая" - астрономы из США увидели бы русских, а те в свою очередь американцев. как кругосветное путешествие из точки A в ту же точку.

аналогией попытался описать именно "плоскую" Вселенную.
как "вектора" (направление обзора, траектория фотона... даже и не знаю как правильно сформулировать). поставь точку, проведи от нее луч, потом из той же точки еще один, перпендикулярный 1ому, потом еще и еще, уменьшая углы между ними. в конце получится круг. а теперь начинай проводить лучи из той же точки,но в других плоскостях. в итоге выйдет сфера. эти лучи/плоскости бесконечны во все стороны("бесконечная Вселенная") но, все зависит именно от той точки, которую ты поставил в самом начале. сдвигая ее (точку наблюдения) будут сдвигаться и все лучи/плоскости. а так, как нет такого понятия "центр" - нельзя выделить какую-то одну приоритетную точку (ну или все точки одновременно являются "центром), то все это замыкается. иными словами "где на земной поверхности центр?"

зы: походу я все еще больше запутал :)
#186 #276132
>>276130
Ок. Когда появится - пиши. Поможем с аналогией.
#187 #276133
>>275847

>не затрагивает(как и красный треугольник на пикче


Почему сделал замечание про треугольник? Разве он принципиален, особенно для рассматриваемого вопроса? По-моему, он просто демонстрирует как одна и та же фигура выглядит в пространствах, имеющих разное искривление.

Что касается вопроса про край, то, насколько я понял из прочитанного, край в случае с незамкнутой вселенной принципиально существует. То есть в определенный момент времени можно быть помещенным в точку дальше, скажем по направлению взгляда, которой континуума нет. НО! Увидеть его нельзя вследствие расширения вселенной, которое происходит, во-первых, - с ускорением, во-вторых,- во все стороны сразу и, таким образом, две точки пространства могут удалятся друг от друга со скоростью, превышающей скорость света (не нарушая закона о максимальной скорости света для физического объекта), что делает невозможным зрительное восприятие такого объекта и сам объект "несуществующим". Более того, вследствие этого расширения перед наблюдателем тут же образуется пространство, где могут быть в том числе макрообъекты, которые попадут туда из соседних областей в ходе расширения пространства. Обосрите.

Вопрос вот в чем - есть ли предел скорости расширения пространства (и как следствие - скорости относительного удаления галактик)?
>>276157
#188 #276137
>>276128

>Вселенная именно незамкнутая. Но при этом бесконечная. Но с кончным числом материи. Вот это я догнать немогу.


Успокойся.
Давай рассмотрим что вселенная "не замкнута". Пишу в оквычках т.к. любое такое определение суть ТЕОРИЯ.

Кароче, Начнем с того что вселеннаая бесконечна потомучто не измеряется в "пространсве которое больше неё самой" потомучто такого пространства нет. Если оно есть - это уже вселенная.

Далее. Конечное число материи - что тебя удивляет? Бесконечное количество материи предпологает бесконечный заряд во вселенной - но энтропия против, и намекает нам что во вселенной дохуя пустоты.

К слову я тоже гуманитарий, мимокрокодильный.
>>276138>>276157
#189 #276138
>>276137
Эмм...как бы, известно, что суммарный заряд вселенной =0
>>276139
#190 #276139
>>276138
Я надеюсь ты не намекаешь что этот ноль равен бесконечности?
>>276140
#191 #276140
>>276139
Нет, конечно.
14 Кб, 500x341
#192 #276145
>>274839 (OP)
Вечер в хату, спейсаны. Я к вам из издача. У нас возникла проблема и лишь вы можете помочь. Сразу скажу, вопрос антинаучный, поэтому ничего серьёзного. Мне нужен ответ на следующий вопрос:
Допустим, у нас на планете возникает сооружение (вопрос его возведения не важен, главное — присутствие), размеры которого выходят за экзосферу. И при этом это единый монолит. Очень большой теоретический блок твёрдого материала.
Теперь сам вопрос: что произойдёт? Этот монолит унесёт в открытый космос? Или что-то случится с самой планетой? Или наступит полный пиздец? Или он может существовать на планете, но будут проблемы из-за отбрасываемой тени? Мне нужен примерный ответ, мы не можем продолжить нашу охуительную повесть. Заранее спасибо.
#193 #276147
>>276145
Его в любом случае распидорасит, солидная часть осколков устроит на Земле ад и израиль. Или у вас условие, что он неуничтожаемый™?
>>276154
#194 #276148
>>276145
"Космический лифт" же, и его варианты.
#195 #276152
>>276145
Рисуй в пейнте.
>>276153>>276154
#196 #276153
>>276152
Зачем ему рисовать, что тебе из описания не понятно? Из земли торчит хуйня высотой более 10 000 км. Можешь представить себе, что это квадратная башня.
>>276156
#197 #276154
>>276147
По сути да. Это цельный брусок из металла, назовём его йобаниум. Если исключить саиый очевидный вариант, что монолит сломается и его распидорасит. Вот именно что будет, если он сохраняет свою целостность, но по сути просто кусок металла. Очень большой кусок металла.

>>276152
Таки просто прямоугольник, выходящий за пределы атмосферы планеты. Никаких особых чертежей.
>>276195
116 Кб, 1437x1437
#198 #276156
>>276153

>Зачем ему рисовать, что тебе из описания не понятно?



Вот это из описания не следует:

>Из земли торчит хуйня высотой более 10 000 км. Можешь представить себе, что это квадратная башня.


Другие размеры, форма, отношение к радиусу самой планеты, плотность материала. Так можно сказать, что и из 67P "торчит монолит".
#199 #276157
>>276133

>Почему сделал замечание про треугольник?



Если вместо треугольника взять круг, получится Наблюдаемся Вселенная с Землей в центре. Если бы Земля была ближе к краю вселенной, и кусочек этого круга выходил бы за его пределы, то с Земли бы этот край выглядел следующим образом: в том направлении на определенном расстоянии галактики вдруг заканчиваются - типа супервоид, за которым ничего нет. Чем ближе Земля была бы к краю вселенной, тем ближе находился бы этот супервоид к нам, вплоть до того, что он занял бы вообще половину Наблюдаемой Вселенной, если бы нам совсем повезло оказаться с самого краешку.

>>276137

>Начнем с того что вселеннаая бесконечна потомучто не измеряется в "пространсве которое больше неё самой" потомучто такого пространства нет. Если оно есть - это уже вселенная.



Так Вселенная же расширяется. Значит увеличивается количество пространства в ней(точно так же как увеличивается оное в надуваемом шарике). Значит пространство тоже конечно. О какой бесконечности втирают ученые?
>>276236>>276239
#200 #276159
Кстати, может такое быть, что Сверхпустота Эридана это и есть край вселенной, каким он видится с Земли?

из вики:
Профессор Университета Северной Каролины Лаура Мерсини-Хафтон считает: «Стандартная космология не может объяснить такой гигантской космической дыры … это явный отпечаток другой вселенной за краем нашей».

Тут, правда, почему-то еще про другую вселенную упоминается...
>>276160
#201 #276160
>>276159
Это бред, бесполезнеж и мудянка. Какой-то альтернативщик протащил шмат говна в педивикию, и его не убирают. И в каждом ТТВ приходится об этом писать.
#202 #276162
Лаура Мерсини-Хафтон - погуглил про неё. Эту милфу частенько показывают в bbc-шных научнопопулярных фильмах. Она из банды приверженцев теории суперструн.
Мультивселенная - это не такая уж альтернативщина, это скорее гипотетическая надстройка над общепринятой наукой, непроверяемая на данном уровне развития науки.

Но если не заглядывать так глубоко, не думать о мутильтивселенной и о других вселенных за пределами нашей, то с точки зрения современной науки и теории большого взрыва Сверхпустота Эридана вполне может быть краем вселенной, не?
>>276163
#203 #276163
>>276162

>непроверяемая


>науки


Наука по определению не оперирует, не имеет дело с нефальсифицируемыми теориями.
>>276200
#204 #276179
Поясните за ядро Юпитера. Почему многие говорят, что оно разрушается? Если это так, то как скажется это на планете?
>>276182
#205 #276182
>>276179

>Почему многие говорят, что оно разрушается?


"Многие" - это кто?
>>276203
#206 #276195
>>276154

>Вот именно что будет, если он сохраняет свою целостность, но по сути просто кусок металла. Очень большой кусок металла.



продавит кору, лава из всех щелей, ядро теряет девственность, за гланды на другой стороне не выходит.
>>276226
#207 #276200
>>276163

>непроверяемая на данном уровне развития


это не означает что эти гипотезы не следует разрабатывать, 100лет назад грав волны тоже были непроверяемыми.
>>276202
#208 #276202
>>276200

>100лет назад грав волны тоже были непроверяемыми


В критерии Поппера непринципиально, можно ли пощупать явление за влажное вымя. Важно другое - можно ли в принципе поставить эксперимент (пусть и мысленный), опровергающий гипотезу/теорию. Поэтому существование грав.волн что 100 лет назад, что сейчас, является научной теорией, а существование мультивселенной - нет. И от уровня развития технологий это не зависит, и зависеть не будет.
>>276206
#209 #276203
>>276182
Почему-то мне кажется, что он про БКП.
#210 #276206
>>276202

>Важно другое - можно ли в принципе поставить эксперимент (пусть и мысленный)


С этим здесь все хорошо
>>276210
#211 #276210
>>276206

>С этим здесь все хорошо


В мультивселенной? Ты не можешь обнаружить что-либо, не принадлежащее этой вселенной, никакими способами. Иначе это уже не другая вселенная, а часть этой. Гипотеза нефальсифицируема.
#212 #276216
>>276210
Как же квантовые компьютеры, которые работают в нескольких вселенных? Шах и мат, аутисты.
>>276223>>276278
#213 #276223
>>276216
Они работает в нескольких черных дырах вообще-то.
#214 #276225
А правда что мы живем в чёрной дыре?
>>276227
#215 #276226
>>276145
Очень мало мы знаем о твоем железном дрыне, друже. Какая у него масса, какая высота? Если он очень массивный, то он может погрузиться в землю под собственным весом, проломить кору и дальше, как сказал >>276195-кун. Если не слишком массивный, то просто заякорится в земле и будет стоять как есть. Если же он легкий и стоит на монолитной поверхности, то он будет неустойчив, как поставленный на торец карандаш, и может грохнуться на землю под влиянием возмущений от той же луны, например.

Еще, если он достаточно высокий, о него будут разъебываться спутники и космические корабли. Не сразу, но со временем он расчистит собой все околоземное пространство. Кстати, в тему вопрос местным умникам: как построить космический лифт и не разъебать о него всю нашу группировку спутников?

Если он будет совсем-совсем высокий, то наверное, его может даже и унести в космос - но для этого его высота должна в несколько раз превышать диаметр планеты.

Зато от тени проблем будет не больше, чем от небоскреба. Ну да, будет по утрам отбрасывать тень на запад, но к полудню все равно солнышко выглянет - и то это если будешь жить под боком. В зависимости от сечения дрына на расстоянии в в 10-20-100 километров он вообще не будет заслонять солнце полностью, и всем будет категорически похуй.

Вот что придумалось навскидку. Местные наверняка поправят, если я наврал.
>>276256
#216 #276227
>>276225
Конечно. Пора заводить трактор.
>>276229
#217 #276228
>>275693
На цифру соседнюю галактику сфотали.
#218 #276229
>>276227
Но куда
>>276230>>276257
#219 #276230
>>276229
В просвещенную Европку из этой дыры ебучей.
>>276313
#220 #276233
Есть гипотеза, что Меркурий когда-то был спутником Венеры. Все ссылки приводят на педевикию, либо на другие говнооколонаучные сайты. Чего скажите по этому поводу?
#221 #276236
>>276157

>О какой бесконечности втирают ученые?


Родной, еще раз...
За пределами вселенной нет измерений. Ну тоесть совсем. Даже если она расширяется, это всёровно не значит что она конечна, т.к. её нельзя измерить в измерении которое не в ней.

А если нет системы измерений больше чем объект измерения - ты не сможешь установить границу - отсюда бесконечость.

Ну тоесть даже если ты и "подойдешь" к этой границе, ты не сможешь её никак зарегистрировать или определить, т.к. "за пределами вселенной" нет ничего, ни пространсва ни времени вобще ничего. Таким образом у тебя нет ориентира чтобы определить конкретную точку как "край вселенной".

И как ты уже сказал - вселенная расширяется. Со скоростю света кстати. Любая информация распростроняется с такойже скоростью. Отсюда опять таки выходит что както зарегистрировать эту границу не представляется возможным в принцыпе, только преодолев эту пресловутую скорость света.

А преодалев - ты сталкнешься с проблемой - см. пункт выше.
Даже если ты сможешь очутиться в противоположных концах одновременно чтобы замерить расстояние между ними, эти самые "концы" тупо невозможно както определить.
#222 #276239
>>276157
>>276236
И вот еще добавочка:

Вот тебе мысленный эксперемент:
Предположим ты начал движение с семли в рандомном напревлении быстрее скорости света.
Первым делом ты обнаружишь что вселенная начинает для тебя "сжиматься".
А дело всё в том что "расширяясь" распростроняется пространство-время, и ты по сути начал движение во времени назад.
Тоесть долетев до "границы" вселенной, ты обнаружишь себя в сингулярности, не имеющей размеров вообще.
>>276240
#223 #276240
>>276236
>>276239
Берем в качестве края вселенной самую крайнюю частицу.
Профит.
>>276280
#224 #276253
>>276210
давай определимся сначала с размерностью нашего континуума
#225 #276254
Во, тут то я и спрошу: допустим я такой беру и гораздо быстрее скорости света пиздячу к альфацентавре. Если я взгляну на землю, то ясен хуй она будет молодеть и за все мое путешествие помолодеет примерно на 4 года. Но если я взгляну на альфацентавру, то по идее я буду раньше видеть события которые там происходили и увижу ее наоборот постаревшей. Где я не прав?
>>276260>>276262
#226 #276256
>>276226

>как построить космический лифт и не разъебать о него всю нашу группировку спутников


просто им придется лучше маневрировать.

А так ставить надо кольцо, а не чистый лифт - помесь лифта и пусковой петли. На высоте в 100-200
#227 #276257
>>276229
MoonL2 там город, неписанной красы, чуваки забубнили, правительства молчат, на бор берут только со справкой.
>>276379
#228 #276259
>>276236
размерность пространства при его масштабировании не изменяется, те это означает что наше трехмерное пространство не расширяется в сторону дополнительного измерения

>И как ты уже сказал - вселенная расширяется. Со скоростю света кстати.


больше, но вообще ты хуйню написал
#229 #276260
>>276254

>Если я взгляну на землю


>Где я не прав


и нихуя не увижу, в сторону центавры надо смотреть, там земляные картинки будут, но само это изображение к земле отношения особенного уже не имеет - так что можно сказать это глюки. Так что лети, за землю не переживай.
>>276262
#230 #276262
>>276254
>>276260
Нихуя он не увидит. Фотоны будут в обратную сторону лететь.
>>276264
#231 #276264
>>276262
Но по прилету на на Центавру я ведь увижу Землю, какой она была 4 года назад?
>>276266>>276316
#232 #276266
>>276264
Как только ты достигнешь скорости света (даже не превысив ее) ты уже пересечешь всю вселенную, мгновенно.
>>276347
#233 #276278
>>276216
блядь, о чем ты? каких "нескольких вселенных"? ученые сами не знают как работает, кек (но работает - факт).

у термина "вселенная" есть два определения: обозримая часть (именно обозримая - т.е если мы ничего не видим дальше - это не означает, что там ничего нет) и ВСЕ существующее.

если в 1ом случае: ты сидишь в комнате без дверей и окон, и утверждаешь: "это ВСЯ вселенная и ничего дальше этих стен не существует", то это всего лишь ограничение в восприятии, твоих координат и прочего. но это не означает, что там за стенкой нет соседа.

2ое определение: если назвать что-то "другой вселенной", то множество "Вселенная" (ВСЕ ЧТО СУЩЕСТВУЕТ), автоматически включит в себя это новое множество "другая Вселенная", т.е расширится

примитивная аналогия: твоя комната - 1ая вселенная, комната соседа - 2ая и т.д., а вместе весь дом - Вселенная (с большой буквы, как самое большое множество, состоящее из других подмножеств более низкого порядка)

капише? а теперь перечитай свое утверждение еще раз и пойми ошибку.
#234 #276280
>>276240
бля... гуманитарий.

и твоя "крайняя точка" становится "центром", а то что было "центром" становится "крайней точкой". нету таких понятий "край", "центр" и т.п. это все субъективно и лишь в головах у людей... так же нет понятий низ, верх, медленно, быстро и т.п. есть лишь одна характеристика "кол-во энергии", а от нее уже и пляшут.

лучше почитай: Мартин Гарднер - Теория относительности для миллионов. сразу придет понимание и отпадут такие вопросы.
52 Кб, 309x296
#235 #276306
Можно ли использовать нагретый газ выделенный из реголита Луны для создания небольшой тяги? То есть представим себе модуль-попрыгунчик на Луне - он забирает небольшое количество пыли, нагревает её, выпускает струю газа, поднимается, пролетает небольшое расстояние, опускается.

Или нет задач?
>>276307>>276317
#236 #276307
>>276306
Ну так примерно могу почувствовать, что на это будет тратиться слишком много энергии и КПД будет никакой.
>>276312
#237 #276312
>>276307
Ну хотя бы пару прыжков он сделать сможет? Это все как альтернатива вертолету на Марсе, который американцы предлагают запустить с Марсровером-2020. Тут небольшую презентацию на эту тему нашел и очень годно обосновывают, что такой вертолет может позволить разведывать местность впереди марсохода, позволяя точнее рассчитывать маршрут.
https://www.youtube.com/watch?v=w3y7iJEe7uM

Вот и на Луне можно было бы такой же попрыгунчик-спутник сделать для запуска вместе с Луноходом.
#238 #276313
>>276230
Что же ты делаешь? Мы же не /po
#239 #276315
Где-то читал, что в космосе бактерии быстрее мутируют, чем на земле под воздействием космической радиации. А могут на МКС внутри космонавтов мутировать какие-нибудь болезни?
Я знаю, что перед отправкой в космос космонавты проходят карантин, но почему нет никакого карантина после миссии? Всех космонавтов со всех сторон окружают люди без масок, обнимают целуют, чуть ли не лижут.
>>276318>>276326
#240 #276316
>>276264
получишь поток фотонов 4летней давности
#241 #276317
>>276306
просто подбирай реголит и бросай ковшом
#242 #276318
>>276315
внутри станции космонавты защищены от космических излучений. если все обстояло бы иначе, и такая защита отсутствовала - то после короткого полета, приземлялись загорелыми, как после какой-нибудь Турции и через некоторое время умирали от лучевой болезни. а нахождение в такой среде длительное время, вообще бы не оставляло шансов вернутся живым.

карантин перед полетом - для того чтобы не изменить... эээ "экосферу" (если такой термин применим) орбиты, солнечной системы и более далеких участков космоса (гугли "панспермия")

а карантин после просто бессмыслен, т.к. оттуда нечего "привезти/подцепить"
>>276336
#243 #276326
>>276315
могут, хотя там и не все так просто, организм человека не очень благоприятная среда для этого.
Должны быть инфицированные носители, своеобразный инкубатор набитые патогенами, если говорить о патогенах.
Радиация на мкс не сильно выше фона, на два порядка вроде. Те если предположить пропорциональное увеличение мутаций - то возможности генерации МКС на уровне 600 человек. Сравни это с 1 миллиардом индусов и негров, и потом вообще со всеми человеками, а потом с промышленными животными, и потом все это вместе с прочей фауной - мизер. На земле как минимум 70ккк инкубаторов - болезни появляются конечно, но периоды измеряются 10летиями.

Относительно бунтов микрофлоры организма, интересный вопрос, в чисто гипотетическом плане, не применительно к мкс, а вообще, мкс в этом плане все также мизер.
Интересный, так как там все еще сложнее).

Но, в этом случае они тогда бы заболели, и я подозреваю что о медицинский мониторинг товарищей на орбите это запалит.
>>276328
#244 #276328
>>276326
хочу спросить у тебя, как у человека, который больше знаком с космобиологией.

верно ли:

>карантин перед полетом - для того чтобы не изменить... эээ "экосферу" (если такой термин применим) орбиты, солнечной системы и более далеких участков космоса (гугли "панспермия")


>а карантин после просто бессмыслен, т.к. оттуда нечего "привезти/подцепить"



или есть какие-то скрытые нюансы ?
>>276336>>276373
#245 #276336
>>276318
Пиздец, а-а-а, пизде-е-ец. Карантин перед полётом оказывается нужен, чтобы не занести жизнь в космос, ну нихуя себе!
А этот уши развесил >>276328 и принял всё за чистую монету. И это спейсач блядь.
#246 #276347
>>276266
Почему тогда свет от Солнца до Земли летит целых 8 с половиной минут? Если с его скоростью можно мгновенно пересечь всю вселенную?
>>276351
#247 #276351
>>276347
Для нас он идет 8 минут, а для фотона это мгновенно. Кароче он сказал немного хуйню, но смысл был в том, что если бы на корабле была кнопка, при нажатии которой корабль мгновенно разгонялся бы до скорости света, то нажать кнопку тормоза, чтобы остановить корабль у альфацентавры просто невозможно, потому что в системе отсчета корабля, с момента как он разогнался до скорости света, не пройдет и доли секунды.
247 Кб, 250x238
#248 #276352
Допустим инопланетная цивилизация хочет скрыть маркеры жизни в атмосфере своей планеты. Это возможно вообще?

В смысле, можно ли какими-то аэрозолями, при помощи колец, или сфер скрыть биогенные-техногенные газы в атмосфере планеты от спектрографа другой инопланетной цивилизации?
>>276394
#249 #276364
Для тех, у кого могут возникнуть вопросы по поводу темной материи.

https://www.youtube.com/watch?v=7BZOloi1iXU
#250 #276373
>>276328
ну относительно космобиологии это ты загнул, к медицине я тоже отношения не имею.

на вот, карантин после прилета http://drugoi.livejournal.com/3998933.html дань паранойи
http://www.utro.ru/news/2003/10/29/245222.shtml

и тд. Не очень меня этот вопрос интересует, но один фиг им реабилитироваться и в это время болеть им явно не стоит, это скорее защита самих космонавтов, чем что то еще.

Перед отправкой, ну было бы тупо отправить больного человека, пусть даже насморком, это раз.
Отсутствие возможности произвести новые штаммы, не означает невозможность микроэпидемии в пределах станции.
Притащить зику или лихорадку на станцию - товарищей которые это допустили, потом с говном сожрут, заебутся отмываться, клеймо будет на надгробии.
Более простые вещи, типо гриппов, это простой работы, который запросто будет стоить лямы баксов.
Опять же какие медицинские процедуры подготовки, что они в себя включают.

Вообщем это профилактика, во избежание нелепой хуйни в первую очередь, а не защита околокосмического пространства.

Защиты всяких марсов, хз на мой субъективный взгляд лютая хуита, если вычесть, что они там жизнь или ее следы искали. Но люди как то обосновывают, это чсв и попил, имеет смысл поискать, что они там пишут.
Имеет ли смысл вообще дергаться на эту тему - не на данном этапе.

Это может стать проблемой при наличии тысяч станций, где люди проживают автономно в больших количествах, миллиарды - тогда да, там придется разрабатывать специальные процедуры на эту тему, на полном серьезе, без паранойи и игрушек.
Но возможно и медицина подтянется к тому времени, и все будет не так печально, как сейчас.
Разморозка вечной мерзлоты и сохранившиеся там штаммы, гораздо более интересная проблема, текущего дня.
#251 #276379
>>276257
Где взять справку
>>276884
#252 #276393
>>276089

>внутри


внутри твоего скафандра
#253 #276394
>>276352
Либо строить кокон вокруг планеты, либо перерабатывать воздух громадными атмосферными комплексами, лбо делать заводы в которых дым не выходит наружу, либо каким-то аэрозолями или химикатами постоянно хуярить во всю атвмосферу, чтоб оно там расщепляло химикаты на менее паливные элементы или чтоб все говно опадало на землю. Но это нереально, думаю.
>>276396
#254 #276396
>>276394
основной признак, с нашей текущей точки зрения, это свободный окислитель в атмосфере, сиречь кислород.

но вообще они там по любому опоздали, это особенно им ничего не дает.
Жизнь у них там скорее существует миллионы лет уже как, до того как они такие параноики вылупились, информация уже ушла.
3957 Кб, 320x180
#255 #276397
Что такое тепловое излучение? Тепло это же типа когда молекулы трясутся и передача тепла когда эти молекулы трясут соседнюю. Но если в космосе есть горячий шар и на неком расстоянии есть тепловой инфракрасный датчик, то как будет передваться тепло? Если больше в космосе нет ни молекулы и нет света - просто два тела в вакууме - шар и датчик.
#256 #276407
>>276397
Молекула трясясь трясет и электроны свои с протонами, а это заряженные частицы. А любые заряженные частицы, трясясь, трясут электромагнитное поле, и вот эта тряска волнами расходится по электромагнитному полю во все части вселенной.
>>276424
#257 #276408
>>276397

Фотонами. Тепло, свет - всё это электромагнитное излучение
>>276424
#259 #276410
Как визуально выглядит поверхность последнего рассеяния? Наши телескопы ведь до неё "добивают"?
#260 #276413
>>276210

>Ты не можешь обнаружить что-либо, не принадлежащее этой вселенной, никакими способами


Косвенно - можно, например на основе оказываемого влияния на нашу Вселенную, вот, например, гипотеза:
https://en.wikipedia.org/wiki/CMB_cold_spot#Parallel_universe
>>276475
41 Кб, 400x600
#261 #276424
>>276407
>>276408
>>276409

Ну ахуеть теперь, тепло - это волна.
>>276440>>276876
#262 #276435
>>276397
Что происходит на гифке?
>>276441
#263 #276440
>>276424
Ээм, не совсем. Тепло - это энергия. Ее передача может происходить как механическим так и волновым путем.
#264 #276441
>>276435
Анон, желавший тайком пробраться на МКС, сделал это, нарядившись в костюм обезьяны. Тем самым напугал астронавтов на МКС.
>>276450
#265 #276444
Мега-гуманитарный тред получился. Что, всех шизоидов тут собрали?
#266 #276450
>>276441
Пруф на это
67 Кб, 230x230
#267 #276470
>>275077

> в трубе под низким давлением


если в трубе на выходе 30% от уровня моря то на входе будет 100% от уровня моря, сообщающиеся сосуды жи.
#268 #276475
>>276413
Если одна "вселенная" оказывает влияние на другую "вселенную", то они уже ни разу не параллельные, а очень даже пересекающиеся.
#269 #276482
Ребятушки подскажите на какой стадии находятся исследования по искревлению пространтсва. Одно время форсилась тема с Пузырем Алькубьерре, но потом затихло.
>>276484>>276821
#270 #276484
>>276482

>на какой стадии находятся исследования по искревлению пространтсва


Кревим патихоничьку, дай бог, чтоб не сломалось.
#271 #276523
>>275036

>Наверно было не один раз, но спрошу: чому не отправляют мусор в космос, на Солнце например? Не обычный мусор, который на свалки вывозят - тут понятно, что пиздец дорого наверно, а всякие там радиоактивные отходы етс.


ссал против ветра?
40 Кб, 819x460
#272 #276549
Это, наверно, платина, но я толком ничего не нагуглил.
Допустим, барицентр Солнце-Юпитер находится на расстоянии от центра Солнца в 750 000 км, но ведь это же средняя величина ? Эта цифра была бы постоянной, если бы Юпитер был один, но еще есть другие планеты, которые влияют на этот барицентр.

Барицентр Земля-Солнце находится на расстоянии 450 км от центра Солнца, получается это уже усредненное значение, которое учитывает влияние всех остальных планет или только с учетом движения одной Земли ? Как, например, меняется этот барицентр, когда все планеты находятся на одной линии с Землей по одну сторону Солнца и наоборот пикрелейтед для наглядности, без учета всяких расстояний и прочего? Это вообще как-то измеряют и отслеживают или нет ?

Как тогда Солнце движется вокруг этого общего центра масс ?
23 Кб, 471x468
#273 #276570
>>276549

>Барицентр Земля-Солнце находится на расстоянии 450 км от центра Солнца


Иногда он находится даже над поверхностью, в разные годы по-разному.

>получается это уже усредненное значение, которое учитывает влияние всех остальных планет


This. В основном Юпитера и немножко Сатурна, остальное в Солнечной системе это мелкий космический мусор.

>Как, например, меняется этот барицентр, когда все планеты находятся на одной линии с Землей по одну сторону Солнца


Они тянут к себе, соответственно барицентр вылезает за пределы Солнца и находится немного над поверхностью.

> Это вообще как-то измеряют и отслеживают или нет ?


Разумеется, было бы странно если нет. Пикрилейтед.
>>276580
#274 #276580
>>276570
>>276549
А, пардон, я в глаза ебусь. ты спрашиваешь про Земля-Солнце отдельно. Но барицентр системы из двух тел - это воображаемая хуйня. Он имеет смысл в матмоделях, и не зависит ни от чего кроме Земли и Солнца. Понятное дело что с реальностью он слабо связан, СС в целом крутится вокруг некоего общего барицентра, образованного планетами, поясами и облаками.
1983 Кб, Webm
#275 #276628
>>276549
хз, может пригодится, масштаб дистанций соблюден, синяя полосочка сила, более светлая скорость, размеры самих тел не совсем в масштабе, логарифм массы чегототам плюс.
426 Кб, 1920x1200
#276 #276663
>>276738
62 Кб, 5096x1584
#277 #276702
Вот придумал. Поставить поверх детектора фотонов светонепроницаемые перегородки, на каждый пиксель. Свет от далёкой звезды попадает только на определённые детекторы на другие детекторы заглушаются перегородками.
И так свет от звезды не попадает в те участки куда попадает свет от планет и они детектятся. Взлетит?
>>276717>>276765
#278 #276717
>>276702

>Свет от далёкой звезды


Ключевое слово - далёкой. Т.е. Угол между лучами от звезды и от планеты будет очень-очень маленький, следовательно решётка и "пиксели" тоже должны быть очень-очень мелкими. Такими мелкими, что тебе дифракция всю картину испортит.
>>276743
#279 #276729
Косманы, слушаю лекции Попова, вот что заметил. Он больше одного раза обговаривал, что если скорость звука в веществе превышает скорость света, то вещество начинает очень плохо себя вести, и он даже не пытается сделать маленькое отступление и на пальцах пояснить, что же случается с веществом. Неужели всё настолько сложно, что без супер матана даже подходить не стоит?
inb4 без матана к физике вообще подходить не стоит
>>276752
718 Кб, 2732x1374
#280 #276738
>>276663
неплохо, годная линка
216 Кб, 1280x800
lvlb !RNCRn8EeGo #281 #276741
Троллфаце
>>276742
134 Кб, 557x676
#283 #276743
>>276717
А если сделать не очень маленький пиксель, а очень длинную перегородку?
#285 #276752
>>276729
Скорость частиц с массой не может быть быстрее света. Звук есть передача энергии через физическое взаимодействие массовых частиц.
#286 #276765
>>276702
1ае на 1пк это 1''
это примерно отношение зазоров к дилнне палочек как 5 к 1 миллиону.
те это 5 милиметров на длинну в 1 км

тоже для 300 световых лет это 5мм на 300 км, примерно.

Делаем по другому
ставим L3 приемник в виде трубки 10 км длинной, диаметром метров 100.
Кванты попадают в трубку, мало не мало но усиливаются
В L4 ставим премник.
Расстояние 260 миллионов километров, газеты на планетах читать можно будет.
>>277568
#287 #276771
кто такие марсиане, если жизни на марсе нет?
#288 #276784
>>275156
Наркоман. Гугли энтазис и курватура. Это делалось, еще аж др. Греками, только не для компенсации кривизны земли, а для компенсации особенностей "оптической системы и ЦП" у тебя в башке.
>>276788
#289 #276785
>>275179
Это про Хоккига они так перевели?
#290 #276787
>>275266
Если нечто с массой будет двигаться с СС, то в повороты оно не сможет.
57 Кб, 1280x720
#291 #276788
>>276784

>курватура

>>276801
108 Кб, 800x788
#292 #276801
#293 #276821
>>276482
Канонично: Пузырь Альбукерке
#294 #276866
Что такое пространство?
11 Кб, 720x524
#295 #276868
>>275077
Эверест это не твой пик, это пикрелейтед.
Это не говоря уже о ебической сложности проведения хоть каких бы то ни было работ на такой высоте. Даже трупы с горы не снимают, а вы хотите ебической сложности архитектурное сооружение собрать.
#296 #276875
>>276742

>Для изменения ориентации спутника достаточно одного точного движения,но спутник вряд-ли выживет с пробитым скафандром.))))


Проиграл.
#297 #276876
>>276424

>Ну ахуеть теперь, тепло - это волна.


Ты не поверишь!11 Но например Солнце, быстро тряся своими протонами, светит нам вполне видимым светом (т.е. электромагнитными волнами).
#298 #276884
>>276379

>Где взять справку


В ближайшем лунянском консульстве, они маскируются под психдиспансеры. Только не забудь надеть шапочку из фольги, перед входом тебя земляне из правительсва будут облучать лучами, чтоб зазомбировать чтобы ты с Земли не убежал.
Только это всё большой секрет, никому не говори!
>>277199
#299 #276917
>>275090
Заготовка для шапки:
Осмиевые тупые вопросы:

>Q: давайте пускать ракету с самолёта


A: Орбита - 8 км/с, а не 100 километров вверх, самолёт, способный в перевозку космической ракеты добавит жалкие 300м/с, это нивелируется малым весом ракеты и усложнением ПН с расчётом на поперечные нагрузки.

>Q: давайте пускать ракету с горы


A: Орбита - 8 км/с, а не 100 километров вверх, потери на преодоление атмосферы в этом числе едва заметны и забарывание этих потерь не стоит ебли с постройкой сложнейшей инфраструктуры на труднодоступной верхотуре.

>Q: давайте пускать ракету с разгонной петли


A: Петля Лофстрома существует только на бумаге и требует дохуя НИОКР и международной кооперации, плюс при её распидорасе её считай надо заново строить.

>Q: давайте ловить фалькон тросами


A: Ракета - это не кузов батиной шестёрки, а тонкая бочка из люминя, которая расчитана на продольные нагрузки и давление внутри. Тросы просто разрежут её нахуй.

>Q: чому фалькон не спускают на парашютах?


A: Нельзя мягко и точно сесть на парашютах. Надо садиться мягко и точно, и это можно сделать с помощью двигателей, что уже доказано на практике.

>Q: чё там в чёрной дыре?


A: Хуй знает.

>Q: космонавты ебутся там на МКС?)0)))0)))0


A: Нет.

>Q: а чё буит если в касмас без скафандра выйти?))00)


A: Задохнёшься, потеряешь сознание и умрёшь.

>Q: а сколька в касмасе градусов?


A: От 3 Кельвинов и выше в зависимости от близости к фонящей хуйне типа Солнца или атмосфере.

>Q: а как делают такие фоточки где млечный путь видно кароч?


A: Большая выдержка.

>Q: м-м-м, терраформация, ахахах, няняня, терраформировать Марс, терраформировать Венеру, терраформировать Энцелад, терраформировать Плутон, терраформировать Землю, терраформировать Юпитер, терраформировать Солнце, терраформировать чёрную дыру (кстати чё там в чёрной дыре?), терраформировать клавиатуру от пекарни, терраформировать мою жопу, терраформировать Аллаха.


A: Когда заселите одновременно Сахару и Антарктиду так, чтобы можно было в любой день года выйти покурить на незастекленный балкон в одних труханах - тогда приходите.

>Q: Как срать в невесомости.


A: На аполлонах срали в мешок и подтирали пукан влажной салфеткой, на МКС используют туалет, который как бы пылесосом какахи засасывает.

>Q: Распидорасит ли человека в открытом космосе без скафандра


A: Нет.

>Q: Почему нельзя быстрее?


>Q: Когда варп-драйв?


A: Можно упасть в пузырь альбукерке, наса уже почти надула его.

>Q: Почему с Земли не фоткают флаг на Луне?


A: С Земли даже толстый спутник на ГСО только точкой виднеется при огромной выдержке, на Луне, которая в 10 раз дальше и движется ты с Земли такую мелочь не разглядишь.
Зато низкоорбитальные лунные спутники уже сфоткали место посадки в меру возможностей оптики.

>Q: Почему мусор не сбрасывают на Солнце?


A: Для того, чтобы упасть на Солнце надо охуенно так оттормозиться, чтобы сойти с орбиты, на которой Земля и мусор (30км/с). Самый шустрый старт был у аппарата New Horizons который набрал "жалкие" 17км/с на старте. Надо строить огромную йобу или использовать гравитационные маневры, а все это очень сложно и в итоге не стоит того.
На эти с ответами не нашёлся.

>Q: Зачем в космос людей посылать?


>Q: Почему планеты по кругу?


>Q: Когда полетим далеко?



Поправки, пожелания, предложения?
инб4 пошёл нахуй, никто шапку не читает, и ответы дебильные
#299 #276917
>>275090
Заготовка для шапки:
Осмиевые тупые вопросы:

>Q: давайте пускать ракету с самолёта


A: Орбита - 8 км/с, а не 100 километров вверх, самолёт, способный в перевозку космической ракеты добавит жалкие 300м/с, это нивелируется малым весом ракеты и усложнением ПН с расчётом на поперечные нагрузки.

>Q: давайте пускать ракету с горы


A: Орбита - 8 км/с, а не 100 километров вверх, потери на преодоление атмосферы в этом числе едва заметны и забарывание этих потерь не стоит ебли с постройкой сложнейшей инфраструктуры на труднодоступной верхотуре.

>Q: давайте пускать ракету с разгонной петли


A: Петля Лофстрома существует только на бумаге и требует дохуя НИОКР и международной кооперации, плюс при её распидорасе её считай надо заново строить.

>Q: давайте ловить фалькон тросами


A: Ракета - это не кузов батиной шестёрки, а тонкая бочка из люминя, которая расчитана на продольные нагрузки и давление внутри. Тросы просто разрежут её нахуй.

>Q: чому фалькон не спускают на парашютах?


A: Нельзя мягко и точно сесть на парашютах. Надо садиться мягко и точно, и это можно сделать с помощью двигателей, что уже доказано на практике.

>Q: чё там в чёрной дыре?


A: Хуй знает.

>Q: космонавты ебутся там на МКС?)0)))0)))0


A: Нет.

>Q: а чё буит если в касмас без скафандра выйти?))00)


A: Задохнёшься, потеряешь сознание и умрёшь.

>Q: а сколька в касмасе градусов?


A: От 3 Кельвинов и выше в зависимости от близости к фонящей хуйне типа Солнца или атмосфере.

>Q: а как делают такие фоточки где млечный путь видно кароч?


A: Большая выдержка.

>Q: м-м-м, терраформация, ахахах, няняня, терраформировать Марс, терраформировать Венеру, терраформировать Энцелад, терраформировать Плутон, терраформировать Землю, терраформировать Юпитер, терраформировать Солнце, терраформировать чёрную дыру (кстати чё там в чёрной дыре?), терраформировать клавиатуру от пекарни, терраформировать мою жопу, терраформировать Аллаха.


A: Когда заселите одновременно Сахару и Антарктиду так, чтобы можно было в любой день года выйти покурить на незастекленный балкон в одних труханах - тогда приходите.

>Q: Как срать в невесомости.


A: На аполлонах срали в мешок и подтирали пукан влажной салфеткой, на МКС используют туалет, который как бы пылесосом какахи засасывает.

>Q: Распидорасит ли человека в открытом космосе без скафандра


A: Нет.

>Q: Почему нельзя быстрее?


>Q: Когда варп-драйв?


A: Можно упасть в пузырь альбукерке, наса уже почти надула его.

>Q: Почему с Земли не фоткают флаг на Луне?


A: С Земли даже толстый спутник на ГСО только точкой виднеется при огромной выдержке, на Луне, которая в 10 раз дальше и движется ты с Земли такую мелочь не разглядишь.
Зато низкоорбитальные лунные спутники уже сфоткали место посадки в меру возможностей оптики.

>Q: Почему мусор не сбрасывают на Солнце?


A: Для того, чтобы упасть на Солнце надо охуенно так оттормозиться, чтобы сойти с орбиты, на которой Земля и мусор (30км/с). Самый шустрый старт был у аппарата New Horizons который набрал "жалкие" 17км/с на старте. Надо строить огромную йобу или использовать гравитационные маневры, а все это очень сложно и в итоге не стоит того.
На эти с ответами не нашёлся.

>Q: Зачем в космос людей посылать?


>Q: Почему планеты по кругу?


>Q: Когда полетим далеко?



Поправки, пожелания, предложения?
инб4 пошёл нахуй, никто шапку не читает, и ответы дебильные
#300 #276918
Шапки не нужны, тем более в таком виде. Вместо помощи только срач разведете.
#301 #276921
>>276920
А я думаю, что потому, что роботы не такие ловкие и умелые, чтобы быстро и качественно эксперименты проводить. И при поломке кто робота чинить будет? А того робота, который чинит робота? Надо дохуя продвинуть робототехнику, чтобы уметь в флотилию самочинящихся роботов.
#302 #276935
>>276917
Да ну эти шапки.
>>277051>>277356
#303 #277044
>>276917

>пошёл нахуй, никто шапку не читает, и ответы дебильные


Собственно да. Не все дебильные, но местами есть.
#304 #277051
>>276935
Прально, лучше в каждм треде отвечать шкальникам че там внутри черной дыры и чому зимой зима.
>>277054
#305 #277054
>>277051
Лучше отвечать школьникам, чем быдло на каждый вопрос посылающее посмотреть хуево сделанную шапку. Мы тут ради общения, а не ответов.
#306 #277195
>>276917
Пошел нахуй со своей шапкой, ебанько.
#307 #277199
>>276884
Мне кажется ты тралишь. Думаешь я лох?
>>277222
#308 #277217
>>276917

>Q: м-м-м, терраформация, ахахах, няняня, терраформировать Марс, терраформировать Венеру, терраформировать Энцелад, терраформировать Плутон, терраформировать Землю, терраформировать Юпитер, терраформировать Солнце, терраформировать чёрную дыру (кстати чё там в чёрной дыре?), терраформировать клавиатуру от пекарни, терраформировать мою жопу, терраформировать Аллаха.


>A: Когда заселите одновременно Сахару и Антарктиду так, чтобы можно было в любой день года выйти покурить на незастекленный балкон в одних труханах - тогда приходите.



хуйня, тут ты свое мнение втираешь.
Лучше уж, вывезите 1000 тонн на орбиту и тогда поговорим об этом.

>Q: Почему все трут за терраформирование, а за космические хабитаты не трут.

49 Кб, 365x543
#309 #277222
>>277199
вообщем смотри что узнал, этот про шапки он короче заливает.
Вообщем узнал, контора Tech tree company, хз странное название, возможно перепеутал. Женег не берут походу работать надо.
Вообщем немного узнал, помогай.
#310 #277275
На многих АМС стоят узконаправленные антенны. После выполнения маневра, насколько я понимаю, КА восстанавливает ориентацию на Землю.
Насколько этот процесс автоматизирован? По идее, должно быть что-то охуительно низкоуровневое и надежное же.
Были ли случаи, когда из-за потери ориентации антенны КА был утрачен (кроме всем известного Викинга)?
>>277301
#311 #277301
>>277275

>Насколько этот процесс автоматизирован?


Полностью, иначе ему не удалось бы восстановить ориентацию, очевидно же. Если ты имел в виду что-то вроде какой-то тупой схемы, которая за это отвечает - обычно такого нет.

>По идее, должно быть что-то охуительно низкоуровневое и надежное же.


В современных КА процесс построения и восстановления ориентации слишком сложен для вынесения в отдельный контур. К тому же единый БЦВК сильно повышает надежность системы, не стоит относиться к нему пренебрежительно, мол слишком высокоуровнево. Смотри сам, с единым БЦВК у тебя всего один жизненно важный контур, а так несколько, пусть и условно не связанных, но сбой в каждом все равно приводит к потере аппарата. Нет смысла их плодить. Обычно выносят в отдельный контур лишь ТМИ, т.к. она не влияет на работоспособность КА или РН.

>Были ли случаи, когда из-за потери ориентации антенны КА был утрачен (кроме всем известного Викинга)?


Фобос-2. Еще недавний Прогресс М-27М, например. Да множество их. Обычно потеря ориентации антенны - это отказ двигателей или маховиков и как следствие потеря ориентации аппарата, а не сбой части СУ (аппаратной или программной), отвечающей за восстановление ориентации после маневра. Такого и не припомню сходу.
>>277307>>277312
#312 #277307
>>277301
Под автоматизацией подразумевал кусок кода, который если что будет в местном аналоге сейфмода самостоятельно пытаться что-то найти. Например, если дебил-погромист выслал инструкцию, в конце которой не прописана ориентация на Землю после маневра. Чтобы не было хуйни как на Викинге.
>>277318
#313 #277310
Есть ли аналитическое решения задачи двух лет в общем виде? Типа летит ракета, известны позиция и вектор скорости, надо найти скорость и позицию через X секунд.
>>277314>>277572
#314 #277312
>>277301

>В современных КА процесс построения и восстановления ориентации слишком сложен для вынесения в отдельный контур


Ок, а что же с аппаратами 60-70-80-х годов? Там ведь из компов - только счеты на электроуправлении. И математически задача ориентации - тривиальна (или нет?). Как там решали, и нельзя ли развернуть корпус/антенну каким-нибудь простым способом?

мимо
>>277318
#315 #277314
>>277310

>Есть ли аналитическое решения задачи двух лет в общем виде?


Ты как, сам, или того?
https://ru.wikipedia.org/wiki/Задача_двух_тел
#316 #277318
>>277307
Вот шлют ли сейчас такую команду вручную - это хз. Собственно сам сейфмод как таковой есть в практически любом современном аппарате, даже в кубсатах, но включается обычно в случае подозрительного отклонения параметров, а не в случае ошибки Земли. В нем бросается выполнение всех рабочих функций и аппарат пытается связаться с Землей.

> Например, если дебил-погромист выслал инструкцию, в конце которой не прописана ориентация на Землю после маневра


Ну например вот случай с Гюйгенсом и Кассини, где забыли включить опорный источник частоты. Не ориентация, но причина та же. Видимо какая-то защита от дурака есть, но от грамотного дурака не может быть спасения.

>>277312

> И математически задача ориентации - тривиальна (или нет?)


Нужно 1) опросить все датчики, 2) посчитать точную команду на включение двигателей (та самая математическая часть), 3) убедиться что ПН и платформа работают в нужных режимах, и только тогда уже включать движки. Все три действия довольно заморочены и алгоритм довольно разветвлён, особенно учитывая что некоторые датчики тоже требуют определенных условий для работы. Можно обойтись без "проца", примитивными средствами, но это будет переусложнено. В 60х-70х были аналоговые СУ, в 80х уже озаботились программируемыми цифровыми комплексами. Да и датчики попроще в 60х были, мягко говоря, и задания попроще, и ПН попроще, и всё попроще.
#317 #277325
Где почитать о рассчетах параметров орбит ИСЗ? Скорость на конкретном участке, время в тени, и всякое такое прочее.
>>277500
#318 #277338

> Как и все субсветовики, он отличался невероятной обтекаемостью корпуса. Ведь космос близок к абсолютному вакууму только с точки зрения объекта, движущегося на малых скоростях. А движение со скоростью, близкой к световой, сродни прорыву через бескрайнюю атмосферную бурю.


Что вы на это скажете?
>>277340>>277407
#319 #277340
>>277338

>со скоростью, близкой к световой


Значение гамма-фактора нужно, иначе это всё ни о чём.
>>277432
#320 #277356
>>276935
Согласен.
#321 #277368
>>277408>>277431
#322 #277407
>>277338
хуйня, давление не высокое, прямо скажем никакущее.

невероятная обтекаемость тоже хуйня, оптимальные формы отличаются для разных условий они различны.

Однако, движение такого шипа в космосе к аэродинамики имеет нулевое отношение, там у него другие проблемы, и он может быть как утюг если это ему поможет уложить системы защиты. И в этом случае можно считать что утюг оптимален, до следующего релиза систем.
#323 #277408
>>277368
павильон
>>277426
#324 #277426
>>277408
Вот жулики!
#325 #277431
>>277368
Васян-светотехник закемарил у софита.
#326 #277432
>>277340
Блядь, ну тупые же вопросы тред, ну.
#327 #277437
>>275179
Я тут кинчик зырил, так там негр сказал что есть такие гигантские завезды что свет от них не смогут поглотить даже некоторые черные дыры, короче пиздатые очень звезды. Еще есть такие завезды которые вместо гелия как наше солнце ебашат в космос кислород.
#328 #277456
>>275597
А почему конструкция с пикрил не сработает? Я не тролл, просто интересно
#329 #277500
>>277325

>Где почитать о рассчетах параметров орбит ИСЗ? Скорость на конкретном участке, время в тени, и всякое такое прочее.



Ключевые слова астродинамика, орбитальная механика.

Вот тут неплохо изложено, кратко и с формулами: http://www.braeunig.us/space/orbmech.htm#position

По-русски вроде Левантовского хвалят, в шапке ксп-треда ссылка есть.

Скорость на конкретном участке = sqrt(GM(2/r - 1/a)), где а = большая полуось орбиты, r = текущая высота ИСЗ, M = масса планеты, G = грав. постоянная. Все расстояния от ЦМ планеты, а не от поверхности.

По поводу затенённости гуглить бета-угол (beta angle).

мимо-задрачиваю-скрипты-в-ксп
#330 #277508
>>277456
возми в руки магнит и железку - и поиграйся - когда тебя начнет сносить, напиши нам, очень важно запомни условия в которых это произошло.
#331 #277523
>>277456

>Потому что нельзя вытащить себя из болота за волосы.


Внешних сил нету же. Как гласит первый закон Ньютона:
Никогда не говори о Ньютоне
Всякое тело продолжает удерживаться в состоянии покоя или равномерного и прямолинейного движения, пока и поскольку оно не понуждается приложенными силами изменить это состояние.
В случае с картинкой сила притяжения между магнитом и машиной заставляет сгибаться и сжиматься палку, которая их держит на расстоянии.
#332 #277557
>>277456
сработает
#333 #277568
>>276765
Что то я твои цифры не понял.
Cинус 1 секунды 0.0000048. Если сделать детектор 100 микрон, а стенки 20 метров, должно всё получиться. Его конечно не повращаешь, но задетектить планету думаю сможет. Технологии вроде доступны.
>>277608
#334 #277572
>>277310

>Есть ли аналитическое решения задачи двух лет в общем виде? Типа летит ракета, известны позиция и вектор скорости, надо найти скорость и позицию через X секунд.


Отвечаю сам себе: нет. Уравнения закрытой формы, выражающего позицию через время, не существует.
#335 #277608
>>277568
100 микрон на 20 метров тонковато, но в принципе возможно. Много чего придется компенсировать, но ок.
другая проблема - шероховатость канала, будут переотражения.
Соответственно надо не просто контролировать чистоту поверхности этого канала, а еще и замутить там какие то полезные шероховатости, и это ебля для дистанции на которой и имеющаяся аппаратура в принципе что то видит.
Уровень ебли допустим тот же, получившиеся возможности гораздо скромнее.

Количество квантов тоже под вопросом, надо считать - может так статься что там 20 лет ждать придется пока чтото попадет - если сравнить с 200мм диаметром - то это в 4кк раз меньше световой поток, а если сравнивать с 10 метрами, то в 8ккк меньше.

Не тут надо как то эксплуатировать оптический резонанс.
ну и все это вне атмосферы - атмосфера все портит.

>Инструменты VLTI обладают точностью наведения на объект около 1 угловой секунды и, что даже более важно, стабильностью ведения порядка 0.2 угловых секунды в час.


Те система слежения уже будет гемор, если на земле.

Короче в космас в космас, все в космас.
>>277632
#336 #277631
>>277628
Нет, с педивикии стянул, там в svg ещё есть если хочешь:
https://commons.wikimedia.org/wiki/File:Local_Group_rus.png?uselang=ru
Насчёт осей - наверное для наглядности сделали.
15 Кб, 936x636
#337 #277632
>>277608
Спасибо.

А если оптически увеличить угол?
>>277756
#338 #277636
Как выглядит пар в космосе?
>>277649
194 Кб, 1280x720
#339 #277649
>>277636
Как пар. Который бурно устремляется в вакуум от места образования. Но он очень быстро отдает тепло излучением в окружающее пространство (т.к. площадь поверхности возрастает на несколько порядков), охлаждается и кристаллизуется в лёд/снег.

Астронавты говорили про сброс мочи за борт, что сначала она моментально вскипает (т.к. давление равно нулю, а при нулевом давлении жидкая вода не существует и тут же обращается в газообразное состояние, т.е. кипит), а потом кристаллизуется. В результате ты видишь в иллюминаторе красивое облачко желтого снега, переливающееся в лучах солнца.

Алсо, выбросы жидкой воды происходят на Энцеладе. Часть оседает льдом назад, часть снежинок улетает в космос.
>>277660>>277753
#340 #277660
>>277649
Спасибо.
Егорка #341 #277670
Что есть бесконечность?
#342 #277671
>>277670

>Что есть бесконечность?


Число, которое больше любого другого.
#343 #277696
>>277670
Отсутствие конца
>>277700
#344 #277700
>>277696
То есть если у тебя больше нету конца, то ты бесконечен? И поделил себя на нуль.
>>277703
#345 #277703
>>277700
Нет. Если твой конец не имеет конца то ты бесконечен
>>277704
#346 #277704
>>277703
Всё имеет свой конец. А сосиска даже два.
123 Кб, 1844x1246
#347 #277753
>>277649

>кристаллизуется в лёд/снег


Насколько я понимаю, если в пределах а.е. или чуть дальше от Солнца, то будет пар, т.к. средняя температура в этих пределах выше 200K.
>>277761>>277773
#348 #277756
>>277632
на вот тебе, https://geektimes.ru/post/269750/ это хорошая штука
хз, не телескопщик

но основная проблема
в том что у тебя не два лучика, а два световых потока и кванты этих световых потоков отличаются
а) спектром
б) направлением, слегка
Потоки не разделены пространственно, в области твоего приемника
На входе ты получаешь фарш из мясорубки, где того что тебя интересует гораздо меньше чем того что не интересует, и тебе требуется выделить полезный материал из этого фарша.

Есть небольшое отличие по вектору и возможно поляризации (хотя сомнительно). Соответственно требуется задавить(заебно) или вытащить полезные моды.
То что ты нарисовал, это по сути обычный оптический телескоп, он делает по сути тоже самое, просто не так как нарисовано у тебя на картинке.
Но хз я не телескопщик, не очень в курсе.
#349 #277761
>>277753
А средняя температура чего? Есть тепловой баланс, это я понимаю. Просто например сферическая капля принимает 1366Вт/м2 с одной стороны, не всей полусферой, т.к. не является АЧТ, и бОльшая часть проходит насквозь или отражает, ибо под углом. А вот излучает уже всей поверхностью. Одно дело здоровая дура с плохим альбедо, вот она может перегреться на солнышке. А у капли равновесие будет сильно ниже нуля по цельсию.
>>277793
#350 #277772
>>277670
Есть оптимальная бесконечность, которая по сути бесконечна, например окружность хотя это с сильной пронзающей небеса натяжкой. Её ещё никто не видел и не знает о ней кроме как в теории.
Есть потенциальная. Та бесконечность что конечна, но её можно бесконечно продлевать. Например числа, линия, луч.
Естественно на практике бесконечность невозможна, кроме окружности пожалуй .Но и она под воздействием внешней среды разрушиться. Или как-то так.
>>277774
#351 #277773
>>277753
Нет. Будет и пар и лед, при образовании пара он забирает у воду большую часть энергии, которую вода восполняет кристаллизацией. От теплоемкостей кристаллизации и парообразования зависит соотношение льда и пара. Я точно знаю, я в восьмом классе такую задачку решал.
>>277793
#352 #277774
>>277772
Чем окружность бесконечней квадрата, я не вдуплил.
>>277775
#353 #277775
>>277774
Квадрат это окружность с углами.
#354 #277793
>>277761
>>277773
Но вблизи Солнца всё же пар будет, разве нет?
Кома - это результат возгонки льда, насколько можно понимать, верно?
>>277811
#355 #277798
Чому не выстрелила концепция большого тупого бустера? Это ж выгодно получается.
>>278041
#356 #277811
>>277793
На уровне орбиты земли будет пар. Дальше есть т.н. водная граница ,точно не помню как называется.
Так же есть метановая, кислородная и т.д. границы.
#357 #277824
>>277813
А что, ноль галактических координат через Андромеду?
94 Кб, 1300x866
197 Кб, 1300x1032
34 Кб, 640x447
#358 #277852
зачем делают волосатые микрофоны?
>>277854>>277875
57 Кб, 604x406
#359 #277853
Как определяют продольные скорости внегалактических объедктов?
Слышал что собираются таковую у Андромеды узнать
>>277863
#360 #277854
>>277852
Что-бы ветер не записывать.
>>277874
#361 #277863
>>277853
Эффект Доплера, например.
>>277873>>277888
#362 #277873
>>277863
Это лучевая.
863 Кб, 2814x2310
#363 #277874
>>277854
почему просто тканью или поролоном не обтянуть?
>>278159
#364 #277875
>>277852
Но причём тут мой ёбаный спейсач? Не припомню АМС с такими микрофонами.
А «волосатый» микрофон это защита под прозвищем «дохлая кошка», снижает уровень шумов (особенно низкочастотных от ветра) на ~25Дб, решает при записи где-нибудь на улице. Всякий паралон и прочие режут меньше шумов и предназначены для студийных условий.
16 Кб, 1206x769
#365 #277888
>>277863
Вот нарисовал.
Как узнать скорость под вопросом?
>>277922
#366 #277922
>>277888
До определённых пределов параллакс помогает, как для далёких галактик считают - не представляю. Может и не считают.

>>277875

>Но причём тут мой ёбаный спейсач


А тут через один вопросы не связанные с космосом.
Точнее они связаны только тем, что Земля тоже находится в космосе, и следовательно всё, что происходит на Земле связано с космосом.
#367 #278023
>>277875

>Но причём тут мой ёбаный спейсач?


При том, что кроме как в этом треде, нигде на этой доске получить ответ на подобный вопрос нельзя. И даже не пытайся заговаривать про сцай.
#368 #278025
ПОясните, если я в космосе возьму лом и загручу его - он будет бесконечно крутиться?
>>278040>>278054
#369 #278040
>>278025
Ну теоретически на него будут действовать внешние силы гравитации а также какое-никакое вещество в пространстве, но тысячу лет наверное сможет крутится.
#370 #278041
>>277798
Говорят что невыгодно.
Но говорят те, кто говорит, что и VTVL первая ступень невыгодна.
Нужен второй Бил, но на данный момент его нет.
#371 #278054
>>278025
На низкой орбите сколько-нибудь большой планеты рано или поздно войдет в приливный захват и остановится. Скорее поздно, чем рано.
#372 #278078
Спейсач, раз уж мы заговорили о крутящихся ломах, то у меня опирающийся вопрос. Допустим еще более мощный йоба-аналог "О, Аллах-частицы". Если у той была кинетическая энергия 48 Дж, то у другой допустим все 200 Дж. И вот эта еба частица попадает на кончик крутящегося лома против его движения-кручения. Перейдет ли вся эта энергия в импульс лому и поменяет его скорость вращения и траекторию, или протоны, даже такие быстрые не взаимодействуют напрямую с макрообъектами?
>>278081
#373 #278081
>>278078
У тебя вопрос по физике за 7 класс.
Передали крутящемуся с частотой Х лому длиной У и массой Й энергию в 200 джоулей на один конец.
Считайте, сучечки.
Дословная цитата из моего школьного учебника физики.
>>278094
#374 #278094
>>278081
Вопрос не в том, на сколько лом длиной х, крутящийся со скоростью у, изменит скорость при взаимодействии с телом, передающим энергию в 200 Дж. А в том, передаст ли протон на релятивистских скоростях свою кинетическую энергию вращающейся железке в космосе, или велика вероятность, что он пройдет насквозь, задев лишь несколько атомных ядер? Есть ведь быстрые нейтроны к примеру, у которых разные показатели проникающей способности.
>>278095
#375 #278095
>>278094
Вероятность есть, особенно если лом из таких плотных элементов как железо или хуже.
Если даже о верхнюю разреженную атмосферу ударяются, то о лом тоже может.
Я примерно так почувствовал, пруфов нет, можете считать меня Петриком.
#376 #278139
>>278098
А зачем поглощать? В землю же.
>>278147
#377 #278147
>>278139
откуда там земля, там бетон
эти хрени если надо подкатил погрузчки - вывез, на утилизацию.
бетон будет разрушаться - замена == капремонт, плюс такой капремон на который только смертников подписывать.

плюс материал - можно выбрать с нормальным сечением захвата
>>278150
#378 #278150
>>278147
Окей, стену не поставил в туннеле, и норм, не?
>>278162
#379 #278158
http://www.astronet.ru/db/msg/1188626

>Лишь у нек-рых звезд угловые размеры диска превышают разрешающую способность крупных телескопов, что дает возможность фотографированием с очень короткими экспозициями "восстановить" изображение звезды.


Анон, это правда? Какие это звезды? Хочу видеть их реальные фото.
#380 #278159
>>277874
ветер в волосах лучше запутывается, чем в поролоне
>>278160
#381 #278160
>>278159
Аэродинамика уровня spc
>>278162
#382 #278162
>>278160
ну так этот >>277875 за поролон, че тут еще добавить.

>>278150
еще хуже, еще меньше контроля за ситуацией.
>>278168
#383 #278168
>>278162
Какой ситуацией? Иногда прогревать мать-сыру землю, big fucking deal. Их угольные электростанции помимо того, что греют землю, воду и атмосферу, так еще и серы с говнами в воздух выбрасывают.
>>278209>>278218
#384 #278169
Много топлива надо чтобы вывести МКС на высокоэллиптическую орбиту пересекающуюся с луной?
#385 #278172
>>278169
Дохуя.
>>278178
#386 #278178
>>278172
Примерно почувствовать можешь?
>>278187
#387 #278182
>>278169
9000 Рогозиных и одна камера.
>>278184>>278197
27 Кб, 600x410
#388 #278184
>>278182

>одна камера


Сгорания?
>>278220
#389 #278187
>>278178
До пересечения с Луной 3260 м/с Δv. МКС имеет массу 420 метрических тонн.

Ракетное уравнение Циолковского: Δv = ve ln(mstart / mend). Получаем нужную стартовую массу: mstart = mend e^(Δv/ve).

Возьмём суперконсервативно и нереалистично водородный движок с эффективной скоростью выхлопа 4500 m/s.

m_start = 420000 kg e^(3260 m/s / 4500 m/s) = ~867 тонн.

Значит нам нужно вывести на орбиту МКС и пристыковать к ней тягач массой как минимум 867 - 420 = 447 тонн. А если взять двигатели пореалистичней, например с выхлопом 3500 m/s, то получается уже под 650 тонн.

И это всё не считая массы двигателей, топливных баков и прочей херни, без которой никуда не улетишь.
>>278193
#390 #278193
>>278187
Так МКС уже на орбите, ее только поднять надо.
>>278195
#391 #278195
>>278193
Ты в огурцач что ли переиграл? Это только там 2к дельты надо для выхода на орбиту и менее 1к дельты для луны, в реале тебе надо более 8к для выхода на орбиту.
Естественно эта цифра для перехода с НОО на лунную.
85 Кб, 480x600
#392 #278197
>>278182

> 9000 Рогозиных и один батут

41 Кб, 600x398
#393 #278209
>>278168
ты троллируешь чтоли ? До 5ТЭв протоны, и не только протоны.
Сам по себе ускоритель существует только потому что у нас нет круче источников высокоэнергетичных ионов. Даже если предположить что ядренбатон можно приспособить для этой хуиты, там и десятой доли такого не будет.
>>278213>>278218
#394 #278211
>>278169
160 тонн
>>278227
#395 #278213
>>278209
Я не он, но я действительно не понимаю, почему эти абстрактные "частицы" не пустить в мать-Землю.
>>278217>>278248
#396 #278217
>>278213
Зачем городить поглотитель и загоняться с его обслуживанием, ели можно всё просто спустить? Да хоть в небо выстрелить, блядь.
>>278248
#397 #278218
>>278209

>5ТЭв


Одна десятимиллионная джоуля.
Я же и пишу >>278168

>big fucking deal


Всем насрать. Мегаджоулями засирается атмосфера, гидросфера и литосфера с помощью электростанций, какая нахуй разница от редкой кончи протонами от гудронного калодёра?
>>278222
#398 #278220
>>278184
Телевизионная.
Так можно и в межзвёздное пространство выйти.
>>278224
#399 #278222
>>278218
Поясни, пожалуйста, что это такое вообще. Радиации наведенной же не будет, так? Материалы только от этой хуйни деградируют. И как оно может "загрязнить" что-то?
В общем, я полное быдло, и не представляю, что по коллайдеру летает, разгоняемое магнитным полем, и чем это грозит.
>>278228>>278254
#400 #278224
>>278220
Бля, а я уже прикидывал, как развернуть массовое клонирование Рогозиных для использования в качестве топлива в реактивных двигателях.
>>278226
#401 #278226
>>278224
В качестве топлива тоже можно, только неэффективно.
Надо на орбиту, и камеру поставить, а то отмазывается, понимаешь, что на земле он.
40 Кб, 368x554
#402 #278227
>>278211
Хоспаде.
>>278229>>278254
#403 #278228
>>278222
Протоны да ионы, въёбываются в материю, фонят повсюду гамма-квантами, нейтронами и прочей хуйнёй. Почти как ядерный взрыв. Только настолько миллипиздрический, что всем насрать.
Регулярно земля бомбардируется частицами с энергиями на порядки превышающими энергии в коллайдере и мы живы, из обезьян вышли и сами стали частицы разгонять. Так что похуй на эти частицы. Главное бошку в трек не совать, а то мозги поджарит слегонца. https://ru.wikipedia.org/wiki/Бугорский,_Анатолий_Петрович
>>278247>>278254
#404 #278229
>>278227
Это керосин-кислород?
#405 #278247
>>278228

>Бугорский,_Анатолий_Петрович


>Закончил МИФИ


Это многое объясняет. Я сам там учился, там все такие распездолы.
#406 #278248
>>278217 >>278213
http://www.youtube.com/watch?v=uWbU2r8CKfo
Не уверен что при таких объемах работ, эта экономия хоть чтото дает, если это вообще экономия.
Обрати внимание на спутниковые фоточки, там поля поля поля, на них выращивают хавку, чтобы ее жрать и продавать соседям.
Если общественность узнает, что придурки не позаботились о безопасности всеми ведомыми и не ведомыми способами, там всех на фарш пустят. Не просто даже узнает, а просто будет иллюзорная зацепка паранойи, которую которую не возможно будет отвергнуть железными фактами.
Я уверен что там и так пикеты были, не дадим родиться черной дыре. Просвещенная европа это конечно вам не пикеты за гору, но тем не менее там свои загоны есть.
Во вторых, не совсем в курсе, что там такое, и мне лень ходить из-за вас дебилов по ссылкам, по которым надо сходить вам.это намек
Но, положение этого уловителя определяется конструкцией агрегата, те возможно вопрос стоял так - вывести хобот тоннель на 10км кудато там, с дополнительным гемороем, или вкорачить эту хрень и не мудиться с лишним тоннелем.
Причем этот тоннель-отвод это не просто дырка в земле, это все еще продолжение элементов удерживающих пучек.
Также многое зависит и от самой местной геологической структуры.

После того как ты напускаешь в сыру землю достаточное количество своих головастиков, там начнуться процессы которые могут не понравиться, в первую очередь рабам которые пашут на твоих плантациях.

Относительно контроля - когда эта мать сыра земля начнет пылить, а она начнет, в том числе радиоактивными изотопами, эта хуита начнет расползаться.
И это пылить она будет в том числе радиоактивными газами.
А этот, в том числе экстренный сброс, критическая часть системы, требующая своего обслуживания и лишние зиверты и ненужные изотопы в прямой кишке, не то что хочется рабочим.

Относительно контроля два - разные элементы и их изотопы - по разному взаимодействуют с с протонами нейтронами. Образуются разные цепочки распадов, среди них есть лучше и хуже - с точки зрения радиоактивного мусора.

В небо выстреливать, это пиздец идея, слава богам хаоса, что ты не имеешь к этому ускорителю ни малейшего отношения, все говно начнет сыпаться на товарищей в 200 км вокруг, и на соседние страны в том числе.
Еще раз, слава богам хаоса.

Напомню, на нем гоняют не только протоны.

Короче тупая идея, и я хз зачем я в это вписался, один хуй я не шарю, и слава богам хаоса, тоже не имею ни какого отношения к БАК.
Но в этой вашей идее так много всего не так, что я не мог остаться равнодушным, в ней просто все пиздецово не так.
Следующий пост по этому вопросу, будет экзамен на знание ссылок.
#406 #278248
>>278217 >>278213
http://www.youtube.com/watch?v=uWbU2r8CKfo
Не уверен что при таких объемах работ, эта экономия хоть чтото дает, если это вообще экономия.
Обрати внимание на спутниковые фоточки, там поля поля поля, на них выращивают хавку, чтобы ее жрать и продавать соседям.
Если общественность узнает, что придурки не позаботились о безопасности всеми ведомыми и не ведомыми способами, там всех на фарш пустят. Не просто даже узнает, а просто будет иллюзорная зацепка паранойи, которую которую не возможно будет отвергнуть железными фактами.
Я уверен что там и так пикеты были, не дадим родиться черной дыре. Просвещенная европа это конечно вам не пикеты за гору, но тем не менее там свои загоны есть.
Во вторых, не совсем в курсе, что там такое, и мне лень ходить из-за вас дебилов по ссылкам, по которым надо сходить вам.это намек
Но, положение этого уловителя определяется конструкцией агрегата, те возможно вопрос стоял так - вывести хобот тоннель на 10км кудато там, с дополнительным гемороем, или вкорачить эту хрень и не мудиться с лишним тоннелем.
Причем этот тоннель-отвод это не просто дырка в земле, это все еще продолжение элементов удерживающих пучек.
Также многое зависит и от самой местной геологической структуры.

После того как ты напускаешь в сыру землю достаточное количество своих головастиков, там начнуться процессы которые могут не понравиться, в первую очередь рабам которые пашут на твоих плантациях.

Относительно контроля - когда эта мать сыра земля начнет пылить, а она начнет, в том числе радиоактивными изотопами, эта хуита начнет расползаться.
И это пылить она будет в том числе радиоактивными газами.
А этот, в том числе экстренный сброс, критическая часть системы, требующая своего обслуживания и лишние зиверты и ненужные изотопы в прямой кишке, не то что хочется рабочим.

Относительно контроля два - разные элементы и их изотопы - по разному взаимодействуют с с протонами нейтронами. Образуются разные цепочки распадов, среди них есть лучше и хуже - с точки зрения радиоактивного мусора.

В небо выстреливать, это пиздец идея, слава богам хаоса, что ты не имеешь к этому ускорителю ни малейшего отношения, все говно начнет сыпаться на товарищей в 200 км вокруг, и на соседние страны в том числе.
Еще раз, слава богам хаоса.

Напомню, на нем гоняют не только протоны.

Короче тупая идея, и я хз зачем я в это вписался, один хуй я не шарю, и слава богам хаоса, тоже не имею ни какого отношения к БАК.
Но в этой вашей идее так много всего не так, что я не мог остаться равнодушным, в ней просто все пиздецово не так.
Следующий пост по этому вопросу, будет экзамен на знание ссылок.
#407 #278254
>>278222

> Радиации наведенной же не будет, так?


Будет и еще как будет. Смысл разгона не в том чтобы ядра дже пидорасить, а протоны на части пидорасить
Его 20 лет сторили и еще 20 лет эксплуатировать будут, и всякие долгосрочные эффекты надо учитывать.

>>278227
че тебе не понравилось обезьянин, прикинул за 10 сек, до того как прочитал этого который считал, и я с ним не согласен, но не сильно мы с ним разошлись.

>>278228

>Регулярно земля бомбардируется частицами с энергиями на порядки превышающими энергии в коллайдере и мы живы, из обезьян вышли и сами стали частицы разгонять. Так что похуй на эти частицы.


При чем тут это, когда вопрос просто в ТБ, и организации дел на производстве, связанном с работой с высоко энергетичными частицами.
>>278262
#408 #278262
>>278254
Так вы, блядь, не стройте коллайдер как Black Mesa, в которой всем поебать на ТБ, а выводите пучок нахуй в пердь гор бельгии и всем будет похуй.
>>278266
#409 #278266
>>278262
трололо, ладно таки запишем, я запомню тебя лодочник
77 Кб, 472x479
#410 #278270
>>278265
Доктор Арзамас-16
#411 #278316
>>278304
Так а хули с ними станет? ОЯТ же хоронят в шахтах, и норм.
>>278362>>278484
20 Кб, 250x188
#412 #278362
>>278316
Когда благословенная Европка превратится во всеевропейский эмират, среди новых хозяев жизни обязательно найдутся трюкачи-приключенцы, которые захотят зарейдить древние гробницы древних жопотрахов и надыбать артефактов на память. Французского-немецкого-голландского они, разумеется, знать не будут, потому что Аллах не велит, поэтому предупреждающих табличек на входе они не прочитают. Наломают ТВЭЛы на кусочки, напихают в сумки, вынесут на поверхность постепенно редея рядами, распродадут на базаре, придут домой и испустят дух, сжимая облученные динары в кулачке. А следом отправятся и их клиенты, и дети, и дети клиентов, и все вокруг вообще. Такое-то проклятье древних кяфиров.

Шутки шутками, но сейчас реально проблема - как обозначить захоронения так, чтобы через тысячу-две лет грядущие поколения, которые вполне вероятно не будут знать ни наших языков, ни наших общепринятых символов, поняли, что:
1. Это очень важное место. Но лезть внутрь не нужно. Даже наоборот, внутрь лезть категорически не нужно.
2. Здесь лежит говно. Очень долговоняющее говно, от которого мы сами не смогли избавиться. Оно воняет не меньше сейчас, чем воняло в нашем времени.
3. Нет, блядь, это не сокровища, а леденящий душу пиздец, который убьет вас и всех вокруг вас! Да, даже спустя десять тысяч лет!

Вот тут интересная статья на тему: http://www.damninteresting.com/this-place-is-not-a-place-of-honor/
#413 #278366
>>278362
Это всё охуительно, и я читал статью сто лет назад, только мы пытаемся понять, чем вреден КОЛИДЕР.
>>278487
#414 #278371
Ребята, а как работает излучение хоукинга? У черной дыры вторая космическая же больше скорости света? Как она излучать может?
>>278374>>278747
#415 #278374
>>278371
Существуют некие парные частицы, которые возникают и тут же друг друга аннигилируют. Они вохникают где угодно. В том числе, вокруг черных дыр. И теоретически возможно, что такая парная частица может возникнуть прямо на горизонте событий: одна половинка снаружи, а другая внутри. Поэтому одну засасывает, а другая от тоски и одиночества начинает фонить. Это и есть излучение Хокинга.
>>278432
#416 #278432
>>278380
>>278374
Спасибо, аноны.
#417 #278439
>>278362

если грядущие поколения настолько тупы, что разучились читать, переводить, конструировать и использовать счётчик гейгера, вообще думать - то они полностью заслужили свои рентген-часы и рак яиц.
>>278444
#418 #278444
>>278439
Ну-ка, читни мне чего-нибудь на староегипетском, неразучившийся. Только чур в розеттский камень не подглядывать!
#419 #278456
>>278362

>Шутки шутками, но сейчас реально проблема - как обозначить захоронения так, чтобы через тысячу-две лет грядущие поколения, которые вполне вероятно не будут знать ни наших языков, ни наших общепринятых символов, поняли...



>сейчас реально проблема


- это сиюминутная надуманная проблема, как попилить деньги, не только на дорогих контейнерах для длительного хранения
>>278388
, но и на привлечении гуманитарных специалистов "кульурологов-футурологов".

>Вот тут интересная статья на тему


Они еще и классный фильм сняли про это (не стебусь, действительно хорошо срежиссирован и снят).
#420 #278472
Зачем астронавты глотают зубную пасту после чистки зубов? Они ебанутые?
https://www.youtube.com/watch?v=vcF5QacG8qA

Им навязывают такие стандарты гигиены что-ли? Наши космонавты выплевывают пасту в салфетки, которые потом сушат.
>>278583
#421 #278484
>>278316
ты бы посмотрел про них, прежде чем пиздеть
#422 #278487
>>278366
мы не пытаемся понять чем он вреден, ничем
пара дебилов пытается понять почему он построен именно так, а как им сниться в их грибных фантазиях.
>>278499
#423 #278499
>>278487
Поясни, нахуя городить поглотитель, и чем грозит облучение Земли-матери на такой глубине?
>>278506
#424 #278506
>>278499
выше было, пидорасом стать можно на кладбище в гробу
>>278531
#425 #278531
>>278506
Нельзя, бля.
>>278538
#426 #278538
>>278531
можно можно, мне дядька сказал
#427 #278545
Зачем зеркало JWST покрыли золотом? Как это влияет на отрожаемый спектр? Как будут вытягивать оптические фоточки в труколоре?
>>278547>>278583
#428 #278547
>>278545

>отражаемый


Не школьник, мамкой клянусь.
#429 #278583
>>278472
Потому, что так проще. Ничего плохого в глотании пасты нет, она не вредна.

>>278545

>Как будут вытягивать оптические фоточки в труколоре?


NEEKAQUE. Он в ИК смотрит.
>>278587
#430 #278587
>>278583
Не разбираешсья - не пизди. Так сложно?
>>278596
#431 #278596
>>278587
Вскукарекнул, и доволен. Где я неправ-то?
Паста не вредная. Астронавтам-то уж точно запретили бы.
Уэбб - ИК-телескоп.
#432 #278635
>>278559
Так если форточку постоянно открывать - сквозняк будет. А простудиться на орбите никто не хочет - работа не ждёт, да и до аптеки 400км вниз. Ещё потом спросят, хули перерасход топлива на Союзе - опять казённый корабль покататься брал, пидор?
#433 #278740
Че за космические паруса, блядь? В космосе же ебаный вакуум.
>>278741
#434 #278741
>>278740

>Че за космические паруса, блядь? В космосе же ебаный вакуум.



Ты что, дефективный?

>https://ru.wikipedia.org/wiki/Давление_электромагнитного_излучения

174 Кб, 573x677
#435 #278747
>>278371

>Ребята, а как работает излучение хоукинга? У черной дыры вторая космическая же больше скорости света? Как она излучать может?


Рождение пары частица-античастица происходит над горизонтом, где вторая космическая меньше скорости света. Одна из частиц упёздывает (разлет идет по (для простоты) круговым орбитам; снимки из пузырьковой камеры/камеры Вильсона в институте видел ведь, пидор, но забыл).
>>278807
#436 #278807
>>278747
А как это связано с теорией, что черная дыра "поглощает" информацию? Ну, мол, черная дыра тнформацию не пидорасит, а делает нечитаемой. Можешь по подробнее про это рассказать, если понял, о чнм я?
>>279040
20 Кб, 317x397
#437 #278959
Чому не работает? Нет, я конечно понимаю почему, но лишь интуитивно.
#438 #279003
>>278362
Во, а ты знаешь о чем говориш. Что там вообще с атомной прикладной наукой происходит? Новые технологии использования энергии деления ядер появляются? Или как диды придумали жечь ураний/плутоний - ошметки закапывать в землю, так все и замерло?
#439 #279040
#440 #279137
>>279132
Много.
#441 #279148
>>279144
Можешь для начала почитать что такое орбита, если не толстый.
#442 #279156
>>279155
Читать я вижу ты не хочешь. Хотя вроде умеешь, раз тут пишешь.
Хоть 1000, без разницы, это все равно упадет обратно через несколько минут.
#443 #279160
#444 #279166
>>279164
Ничего. Можешь начинать строить ракету из говна и палок, способную разогнать себя на 8 километров в секунду.
Если бы ты знал физику хотя бы на уровне 9-го класса, я бы тебе посоветовал посчитать потребную энергию, но ты видно из тех, что отрицают законы физики.
#445 #279169
>>279167
Да ничего тебе не мешает, у тебя наверное в гараже стоит сверхзвуковой самолет из фанеры, а печку ты топишь урановыми дровами.
Хочешь напихать в ящик фейерверков и поджечь, надеясь, что это улетит в космос, я правильно понимаю?
>>279174
#446 #279172
>>279170
Гениально. Узнал бы хоть, что порох не требует кислорода извне для горения. И какой удельный импульс у порохового двигателя, как сложно добиться устойчивого горения на всем протяжении работы двигателя, и что это требует.

Еще из физики - формула Циолковского. Позволит посчитать, насколько разгонится ракета с заданной тягой и запасом топлива. Для пороха цифры тебя неприятно удивят.

Ты задаешь такие вопросы, что у меня два варианта - либо ты еще школьник классе в 8-м, и просто не проходил эти темы и мечтаешь о собственной ракете, либо у тебя нет никакого технического образования, даже в рамках школьной программы. Совет - читай книжки, благо есть такие, что даже даун поймет. И поиграй в KSP, чтобы понять, что даже в аркадной игре вывести ракету на орбиту (а первая космическая там всего 2.5км/с), тем более на бочках с порохом - задача нетривиальная.
>>279200
#448 #279175
>>279173
Пихай. Станешь первым в мире человеком, построившим из фейерверков космическую ракету. Просто же хуле, просто пихаешь фейерверки кучей, и оно улетает в космос. Можно еще джве ступени сделать, чтобы как у взрослых дядей.
Хотя мы все прекрасно понимаем, что дальше кукареканья эти безумные эксперименты не зайдут

>>279174
Очень легко перепутать троллинг с реальной тупостью. Троллить тупостью также тупо как быть тупым.
53 Кб, 397x392
#449 #279176
Если растопить лед с астероидов или далеких планет, то эту воду человеку можно пить или нет?
>>279177>>279179
#450 #279177
>>279176
Можно, разрешаю.
>>279178
#451 #279178
>>279177
Как то слишком легко ты даешь согласие. Как бы потом в морге не оказаться после глотка такой водички.
#452 #279179
>>279176
добавь ток солей и миниралов - и вперёд.
С астеройда она дистелят чаще всего.
>>279201
#453 #279180
Почему ракета не улетает во время Static fire test?
>>279219
#454 #279200
>>279172

>что порох не требует кислорода извне для горения


Воу-воу, получается ничто не мешает стрелять в вакууме из обычного огнестрела?
>>279264>>279275
#455 #279201
>>279179
А почему нельзя дистиллированную упарывать? А необходимые соли в таблетках жрать? По-моему ничто не мешает.
441 Кб, 1280x1024
#456 #279219
>>279180
Потому что бросила якорь.
#457 #279231
>>274839 (OP)
Почему челябинский метеорит не заметили до его падения?
>>279239
#458 #279235
>>275090

>а сколька в касмасе градусов?


В СМЫСЛЕ, там же нехуй измерять, вакуум
#459 #279239
>>279231
Слишком мелкий, тусклый, быстрый.
279 Кб, 801x495
#460 #279250
Луна же медленно удаляется от Земли, так? Что будет с Луной после того, как она удалится от Земли через миллиарды лет? На какую орбиту она выйдет? Ну, если забыть про эволюцию Солнца в красный гигант.
>>279597
#461 #279264
>>279200
Мешает. Нужна специальная смазка и материалы для работы оружия при сверхнизких температурах. Причём материалы должны быть с минимальным коэффициентом теплового расширения.
А так порох будет гореть-взрываться
>>279305
#462 #279268
Телескоп Вебба сможет окончательно доказать или опровергнуть существование/отсутствие планет у ближайших звезд? Просто накой хер нам наблюдать планеты в 1000 световых годах от нас? Важно есть ли они у ближайших звезд, ближе 10 световых лет.
36 Кб, 640x240
#463 #279275
>>279200
Я тебе больше скажу, однажды даже стреляли.
>>279310
#464 #279305
>>279264

>материалы для работы оружия при сверхнизких температурах


Ээм, нахуя? Про коэффициент теплового расширения понятно. Но можно модный кожух с водой захуячить, как дедушка Хирам Максим завещал. Тип водяное охлаждение и вся хуйня. А сверхнизкие температуры откуда?
>>279395
#465 #279310
>>279275
Да вот я и спрашивал все же про обычный огнестрел. Просто помню после выхода CoD: ghosts все хуесосили стрельбу на орбите. Я думал, что невозможно по причине отсутствия окислителя, а тут такое узнается, что порох полностью укомплектован. Вот и думаю, а может все же можно. Просто логика какая. Боек бьет в капсюль, порох детонирует, разогревает газы внутри гильзы, давление повышается, пуля покидает гильзу и вылетает из дула, и так же следующая. Что мешает делать это без всяких приготовленийи охуевать от отдачи в безумной карусели?
>>279314>>279321
#466 #279314
>>279310
На том пикрилейтеде - самый обычный огнестрел с минимальной доводкой. Сняли с бомбардировщика, прикрутили к Алмазу, даже разок пульнули в пустоту, чтоб посмотреть что будет.
#467 #279321
>>279310
низкие температуры горению пороха не помеха(до азота как минимум), с механической частью могут быть проблемы, хрупкость металлов, тепловые расширения-сжатия, смазка замерзла, сварка деталей.
Но все это относительно легко устраняется.
#468 #279334
Привет, анон! Какой интересный, небанальный вопрос задать космонавту на конференции? Не могу придумать.
>>279336>>279910
#469 #279336
>>279334
Спроси у него, что бы он сделал, если бы оказался на Луне. Спроси различимы ли созвездия с МКС, или там столько звезд что ничего не понятно. Спроси следят ли они за планетами с МКС например, есть ли у них там телескоп, а не просто камеры. Спроси видят ли они метеоры попадающие в атмосферу земли. Спроси нет ли экспериментов с комарами в невесомости. Спроси отличаются ли выходы в открытый космос от тренировок под водой и как сильно. Кучу вопросов можно придумать.
>>279357>>279375
#470 #279357
Вот этот >>279336 возбудил в моей голове вопрос:
А нельзя ли прихуячить к МКС какой-нибудь телескоп?
>>279362>>279375
#471 #279362
>>279357
Можно, к другим станциям прихуячивали в своё время.
1918 Кб, 4284x2844
#472 #279375
>>279336

>есть ли у них там телескоп, а не просто камеры


>>279357

>А нельзя ли прихуячить к МКС какой-нибудь телескоп?


Нормальный - нельзя, ибо космонавтики топают и колышут станцию (по этой причине хотят запилить свободнолетящий компаньон станции Ока-Т2), да и непонятно как быть с ориентацией.

Хотя некоторые элементы обсерватории на станции есть, например
https://en.wikipedia.org/wiki/Alpha_Magnetic_Spectrometer
https://en.wikipedia.org/wiki/SOLAR_(ISS)
Прямо такого чтобы смотреть - нету, хотя у них там на станции есть такие йоба-объективы, что вполне за небольшие телескопы сойдут. Алсо, из ненаучного есть куча визиров на модуле Заря - ВШТВ (визир широкоугольный точной вертикали, пикрилейтед), ПУМА, визир пилота и т.д. Они - пережиток прошлого, когда в дополнение к датчикам требовалась возможность построить ориентацию вручную, как когда-то на Аполлонах.
#473 #279395
>>279305

> >материалы для работы оружия при сверхнизких температурах


> Ээм, нахуя? Про коэффициент теплового расширения понятно. Но можно модный кожух с водой захуячить, как дедушка Хирам Максим завещал. Тип водяное охлаждение и вся хуйня. А сверхнизкие температуры откуда?


От верблюда. На солнце будет охуенно нагреваться, в тени охлаждаться почти до абсолютного нуля. То есть если орудие будет хоть чутка сложнее однозарядного самопала нужно чтобы элементы сохраняли свои свойства в широком диапазоне температур иначе например в тени порвет ствол на солнце заклинит автоматика.
>>279397>>279402
#474 #279397
>>279395

>На солнце будет охуенно нагреваться, в тени охлаждаться почти до абсолютного нуля.


ИСЗ не успевают охладиться на низких орбитах, период слишком мал. Даже при β = 0° не успевают. Да собственно ни при каких не успевали бы, например в точке L2 можно охладиться лишь до 80К, при наличии тепловых зонтиков до 30-40К (даже JWST с собой несет запас жидкого гелия для охлаждения).
#475 #279402
>>279395

>в тени охлаждаться почти до абсолютного нуля


Ой, все.
>>279438
#476 #279403
>>279397
Алсо, добавлю - в целом тепловой баланс любого КА вполне настраивается при проектировании так, чтобы ММГ (обычно) не замерзал и не испарялся, например. Т.е. вполне жизнеспособный диапазон температур, грубо говоря от -10 до +60 градусов.
#477 #279415
>>279397
Поясни плез, что такое бета-угол. Я нихуя не понял из английской вики.
Наклонение плоскости орбиты относительно плосткости, в которой лежит Солнце и Земля?
>>279419
#478 #279419
>>279415
Отклонение отбиты от направления на солнце. Если оно нулевое, то крафт максимум времени проводит в тени, если 90, то все время жарится.
>>279427
#479 #279427
>>279419
Картинка хуёвая там.
Т.е. 90 - когда плоскости орбит перпендикулярны, так?
А что если у нас такое наклонение и орбита, что крафт в самой дальней от солнца точке выходит из тени планеты? Как это считать для не круговых орбит?
>>279453
#480 #279438
>>279402
Да пошёл ты нахуй животное
>>279454>>279516
#481 #279453
>>279427
это не идеальная метрика, практическая ценность явно для качественно оценки на низких и круговых.
#482 #279454
>>279438
что так резко, вон этот >>279397 написал
#483 #279516
>>279438
Манька серенькнул под себя и теперь, сидя в говне, лает на прохожих,
#484 #279547
>>275114
>>275118
Скорость тьмы, наверно, может быть сверхсветовой. Вот солнечный зайчик (точка от лазера) может быть сверхсветовой, вот она блеснула, хуяк, и сразу тьма, значит, ее скорость не меньше, чем у солнечного зайчика.
#485 #279591
Может не туда пишу, но такой вопрос, ведь чем глубже в воде, тем более у неё давление на саму себя, так?
Т.е. на 1км в глубину океана давление выше, чем на поверхности воды.
А насколько сильно это давление?
И, суть, а изменяются ли характеристики воды на очень больших глубинах? В той же марианской впадине? Она там не становится твёрдой, например?
>>279596>>279913
280 Кб, 1110x1801
#486 #279596
>>279591
Обратимся к педивикии:
У дна Марианской впадины давление воды достигает 108,6 МПа, что примерно в 1072 раза больше нормального атмосферного давления на уровне Мирового океана.

>Она там не становится твёрдой, например?


Жак Пикар и Дон Уолш недоумевая смотрят на тебя со дна Марианской впадины сквозь минувшие с их первого погружения 56 лет.
>>279599
#487 #279597
>>279250
Должна стать спутником солнца же
#488 #279599
>>279596
Я не совсем про это.
Т.е как я понял на где давление выше, там болье частиц Н2О на единицу пространства, так?
А гдеогромное давление вода свойства свои не меняет?
Я читал что в Юпитере Гелий или Водород в металлическом состоянии, например.
#489 #279605
>>279599
То юпитер, а то сраных 11км под водой на земле. Для этого нужно YOBA давление.
>>279608
#491 #279608
>>279605
Т.е. на дне морском хуйня, а не давление и ничего не меняется? Жаль.
>>279612
73 Кб, 600x600
#492 #279612
>>279608
Ну как хуйня, тебе бы точно мало не показалось.
>>279615
43 Кб, 644x594
24 Кб, 600x600
16 Кб, 500x500
39 Кб, 650x865
#493 #279615
>>279612
Алсо, там прямо сейчас плавают вот такие няши, от которых мне криповатенько.
>>279785
#494 #279631
>>279599
Вода оче слабо сжимается. Даже под такими большими давлениями кубометр морской воды остается массой приблизительно в тонну.
#495 #279633
Получается на дне морском сверхкритическая жидкость?
>>279639
#496 #279639
>>279633
Что привело тебя к такому выводу? Температуры там вполне околонормальные.
>>279646>>279652
#497 #279646
>>279639
А если допустим грунтового дна нет? Вода будет оставаться жидкой глубже 100км или 1000км? Такое может быть на какой-нибудь планете-океане.
>>279647
#498 #279647
>>279646
Если давление водного столба на глубине N превышает критическое для перехода в другое фазовое состояние на диаграмме, и температура соответствующая, то оно просто переходит в это состояние. Всё просто. для того что ты хочешь нужен жар и многие тысячи км глубины

> планете-океане


Согласно современным представлениям, такая планета образоваться не может.
>>279658>>280165
#499 #279652
>>279639
Может где на плато и нормальные, но во впадене вулканы постоянные, и возе них достаточно сверхкритично.
>>279662
#500 #279658
>>279647
А что препятствует образованию такой планеты? Все это ГИПОТЕЗЫ, не забывай, также как и факт существования таких планет.
#501 #279662
>>279652
Может быть. Может быть там, где подходящие условия и есть сверхкритическая вода. Вот только что с того? Вода не сдетонирует энивей.
#502 #279771
Думая о всяких сверхкритичностях, подумалось. А может серия КА Венера приземлялась в низины, А то что нам кажется горами на деле острова.
>>279802
#503 #279785
>>279615
Вытащи их на поверхность и посмотри, как их распидорасит.
#504 #279802
>>279771
Ну вот еще подумай и реши. Сидят дядьки, некоторые из них даже горят идеей, и выдвигают гипотезы на основе полученных данных. Выдвигают и отвергают. Неужели ты думаешь, что вот сейчас, в 2016 году ты, анонимус из спц, решил внезапно, подумав, какую-то загадку природы, которую никто даже не додумался загадать?
#505 #279909

>>279301


>Я читал


А я зато про барона Мюнхгаузена читал.
#506 #279910
>>279334

>Привет, анон! Какой интересный, небанальный вопрос задать космонавту на конференции? Не могу придумать.


Спроси его, почему пространство-время считается континуумом.
#507 #279913
>>279591
Жидкость практически несжимаема. Гидравлические прессы, закон Паскаля, вся хуйня, помнишь? Насчет свойств не знаю, но, думается, ей посрать.
#508 #279969
Почему союзы и прогрессы не стыкуются при помощи АПАСа? Вроде бы ж такая годнота была по описанию: и периферийный, и андрогинный, и агрегат стыковки. Вроде бы, и шаттлы им пользовались, и китайцы вон тоже что-то подобное запилили. И сейчас НАСА пилит свой вариант на будущее. А мы сами придумали, сами нос воротим.
>>280907
43 Кб, 400x555
#509 #280165
>>279647

>Согласно современным представлениям, такая планета образоваться не может


Ну хуй знает. Звезды из чистого кислорода тоже никто не ожидал, но она есть.
>>280278
#510 #280278
>>280165
Перекат
#511 #280907
>>279969
Вангую, в конце восьмидесятых уже наверняка в полной мере стал вопрос о "нетуденех", а внедрить АПАС в серийные союзы да прогрессы и модули станций - те ещё денежки.
ПЕРЕКОТ #512 #281001
Перекот имени, найди два отличия
>>280996 (OP)

https://2ch.hk/spc/res/280996.html (М)
Тред утонул или удален.
Это копия, сохраненная 3 сентября 2016 года.

Скачать тред: только с превью, с превью и прикрепленными файлами.
Второй вариант может долго скачиваться. Файлы будут только в живых или недавно утонувших тредах. Подробнее

Если вам полезен архив М.Двача, пожертвуйте на оплату сервера.
« /spc/В начало тредаВеб-версияНастройки
/a//b//mu//s//vg/Все доски